Формула реактивного сопротивления конденсатора: Недопустимое название | Викитроника вики

Содержание

Расчёт реактивного сопротивления конденсатора и индуктивности. Он-лайн калькулятор сопротивлений ёмкости Xc и индуктивности Xl переменному току.

Прежде, чем мы приступим к расчётам разнообразных пассивных и активных фильтров, не плохо было бы сориентироваться в пространстве и задуматься — а за счёт чего происходит процесс частотной фильтрации сигналов, какой неведомый зверь должен выбежать на свист царевича после преобразования частотно-зависимыми цепями, и что это за цепи такие — частотно-зависимые?

Большая Энциклопедия Нефти и Газа учит нас, что частотно-зависимыми цепями называются электрические цепи с использованием емкостных и резистивных элементов. Спасибо, господа нефтяники и газовики — будем знать. От себя добавлю, что индуктивные элементы в частотно-зависимом хозяйстве также иногда пригождаются.

Для постоянного тока ни конденсаторы, ни катушки индуктивности никакого интереса не представляют. Сопротивление идеального конденсатора — бесконечность, индуктивности — ноль. Другое дело — переменный ток, тут наши частотно-зависимые элементы, начинают приобретать определённые значения сопротивлений, называемые реактивными сопротивлениями. Ясен пень, значения этих сопротивлений зависят от частоты протекающего тока. Для особо продвинутых, вымучаю из себя умную фразу — «Реактивное сопротивление – электрическое сопротивление переменному току, обусловленное передачей энергии магнитным полем в индуктивностях или электрическим полем в конденсаторах».

Графики, фазовые сдвиги, интегралы и прочие атрибуты студенческих знаний, как правило, мало кого интересуют. Если я не прав, пусть первыми бросят в меня камень и с лёгкостью найдут необходимую информацию на других сайтах. А мы ребята весёлые, поэтому сразу перейдём к делу и напишем всего пару формул:

Xс = 1 / 2πƒС,   Xl = 2πƒL, где
Xc — сопротивление конденсатора переменному току, а Xl — сопротивление индуктивности переменному току.

РИСУЕМ ТАБЛИЧКУ ДЛЯ РАСЧЁТА РЕАКТИВНОГО СОПРОТИВЛЕНИЯ КОНДЕНСАТОРА

ТО ЖЕ САМОЕ ДЛЯ РАСЧЁТА РЕАКТИВНОГО СОПРОТИВЛЕНИЯ ИНДУКТИВНОСТИ

В реальной жизни конденсаторы, помимо ёмкости, обладают также собственными последовательным и параллельным сопротивлениями и индуктивностью, а катушки индуктивности — омическим сопротивлением провода обмотки и межвитковой паразитной ёмкостью.

Нужно Вам вооружаться этими знаниями, или нет, судить не возьмусь, а вот то, что электролитические конденсаторы имеют обыкновение иногда взрываться при превышении допустимых уровней напряжений, либо перегреве, вызванным утечками вследствие старения — знать надо обязательно.
Делают они это, ни кем не посоветовавшись, эффектно, громко, с выделение токсичных паров электролита в виде облака из дыма, и с лёкгостью могут выбить глаз пытливому радиолюбителю.
Так что, если не хотите превратиться в одноглазого шахматиста из Васюков, соблюдайте технику безопасности, покупайте электролиты приличных производителей.

 

Сопротивление конденсатора

Господа, сегодняшнюю статью можно считать в некотором роде продолжением предыдущей. Сначала я даже хотел поместить весь этот материал в одну статью. Но его получилось довольно много, на горизонте были новые проекты, и я в итоге разделил его на две. Итак, сегодня мы поговорим про сопротивление конденсатора переменному току. Мы получим выражение, по которому можно будет рассчитать, чему равно сопротивление любого конденсатора, включенного в цепь с переменным током, а в конце статьи рассмотрим несколько примеров такого расчета.

Сразу оговорюсь про одну важную вещь. Вообще говоря, реальный конденсатор обладает помимо емкостного

сопротивления еще резистивным и индуктивным. На практике все это надо обязательно учитывать, потому что возможны ситуации (обычно связанные с ростом частоты сигнала), когда конденсатор перестает быть конденсатором и превращается… в некое подобие катушки индуктивности . При проектировании схем этот момент обязательно надо иметь в виду. Согласитесь, господа, крайне неприятно поставить в схему конденсатор и потом столкнуться с тем, что из-за высокой частоты он ведет себя и не как конденсатор вовсе, а как самый настоящий дроссель. Это, безусловно, очень важная тема, но сегодня речь пойдет не о ней. В сегодняшней статье мы будем говорить непосредственно про емкостное сопротивление конденсатора. То есть мы будем считать его идеальным, без каких бы то ни было паразитных параметров вроде индуктивности или активного сопротивления.

Давайте представим, что у нас есть конденсатор, который включен в цепь с переменным током. В цепи больше нет никаких компонентов, только один конденсатор и все (рисунок 1).

Рисунок 1 – Конденсатор в цепи переменного тока

К его обкладкам приложено некоторое переменное напряжение U(t), и через него течет некоторый ток I(t). Зная одно, можно без проблем найти другое. Для этого надо всего лишь вспомнить прошлую статью про конденсатор в цепи переменного тока, там мы про все это довольно подробно говорили. Будем полагать, что ток через конденсатор изменяется по синусоидальному закону вот так

В прошлой статье мы пришли к выводу, что если ток изменятся вот по такому закону, то напряжение на конденсаторе должно меняться следующим образом

Пока что ничего нового мы не записали, это все дословное повторение выкладок из предыдущей статьи. А сейчас самое время их немного преобразовать, придать им чуть другой облик. Если говорить конкретно, то нужно перейти к комплексному представлению сигналов! Помните, на эту тему была отдельная статья? В ней я говорил, что она нужна для понимания некоторых моментов в дальнейших статьях. Вот как раз и наступил тот момент, когда пора вспомнить все эти хитрые мнимые единицы. Если говорить конкретно, то сейчас нам потребуется

показательная запись комплексного числа. Как мы помним из статьи про комплексные числа в электротехнике, если у нас есть синусоидальный сигнал вида

то его можно представить в показательной форме вот так

Почему это так, откуда взялось, что здесь какая буковка значит – обо всем уже подробно говорили. Для повторения можно перейти по ссылке и еще раз со всем ознакомиться.

Давайте-ка теперь применим это комплексное представление для нашей формулы напряжения на конденсаторе. Получим что-то типа такого

Теперь, господа, я хотел бы вам рассказать еще про один интересный момент, который, наверное, следовало бы описать в статье про комплексные числа в электротехнике. Однако тогда я про него как-то позабыл, поэтому давайте рассмотрим его сейчас. Давайте представим, что

t=0. Это приведет к исключению из расчетов времени и и частоты, и мы переходим к так называемым комплексным амплитудам сигнала. Безусловно, это не значит, что сигнал из переменного становится постоянным. Нет, он все так же продолжает изменяться по синусу с той же самой частотой. Но бывают моменты, когда частота нам не очень важна, и тогда лучше от нее избавиться и работать только с амплитудой сигнала. Сейчас как раз такой момент. Поэтому полагаем t=0 и получаем комплексную амплитуду напряжения

Давайте раскроем скобки в экспоненте и воспользуемся правилами работы с показательными функциями.

Итак, у нас имеется три множителя. Будем разбираться со всеми по порядку. Объединим первые два и запишем выражение следующего вида

Что мы вообще такое записали? Правильно, комплексную амплитуду тока через конденсатор. Теперь выражение для комплексной амплитуды напряжения принимает вид

Результат, к которому мы стремимся, уже близок, но остается еще один не очень приятный множитель с экспонентой. Как с ним быть? А, оказывается, очень просто. И снова нам на помощь придет статья по комплексным числам в электротехнике, не зря ж я ее писал . Давайте преобразуем этот множитель, воспользовавшись формулой Эйлера:

Да, вся эта хитрая экспонента с комплексными числами в показателе превращается всего лишь в мнимую единичку, перед которой стоит знак минус. Согласен, возможно, осознать это не так просто, но тем не менее математика говорит, что это так. Поэтому результирующая формула у нас принимает вид

Давайте выразим из этой формулы ток и приведем выражение к виду, соответствующему закону Ома. Получим

Как мы помним из статьи про закон Ома, у нас ток равнялся напряжению, деленному на сопротивление. Так вот, здесь практически то же самое! Ну, за исключением того, что у нас ток и напряжение – переменные и представлены через комплексные амплитуды. Кроме того, не забываем, что ток течет у нас через конденсатор. Поэтому, выражение, которое стоит в знаменателе, можно рассматривать как

емкостное сопротивление конденсатора переменному току:

Да, выражение для сопротивления конденсатора имеет вот такой вот вид. Оно, как вы можете заметить, комплексное. Об этом свидетельствует буковка j в знаменателе дроби. А что значит эта комплексность? На что она влияет и что показывает? А показывает она, господа, исключительно сдвиг фаз в 90 градусов между током и напряжением на конденсаторе. А именно, ток на 90 градусов опережает напряжение. Этот вывод не является для нас новостью, про все это было подробно рассказано в прошлой статье. Чтобы это лучше осознать, надо теперь мысленно пройтись от полученной формулы вверх к тому моменту, где у нас это

j возникло. В процессе подъема вы увидите, что мнимая единица j возникло из формулы Эйлера из-за того, что там был компонент . Формула Эйлера у нас возникла из комплексного представления синусоиды. А в исходной синусоиде как раз был заложен сдвиг фазы в 90 градусов тока относительно напряжения. Как-то так. Вроде все логично и ничего лишнего не возникло.

Теперь может возникнуть два совершенно логичных вопроса: как работать с таким представлением и в чем его выгода? Да и вообще, пока лишь какие-то дико абстрактные буковки и нифига не ясно, как взять и оценить сопротивление какого-нибудь конкретно конденсатора, который мы купили в магазине и воткнули в схему. Давайте разбираться постепенно.

Как мы уже говорили, буковка j в знаменателе говорит нам лишь о сдвиге фаз тока и напряжения. Но она не влияет на амплитуды тока и напряжения. Соответственно, если

сдвиг фаз нас не интересует, то можно исключить эту буковку из рассмотрения и получить более простое выражение абсолютно без всяких комплексностей:

Согласитесь, жить стало чуточку легче. Это выражение позволяет рассчитать сопротивление конденсатора для конкретной емкость и частоты сигнала. Заметьте, господа, интересный факт. Сопротивление конденсатора, оказывается, зависит не только от самого конденсатора (а именно его емкости), но и от частоты протекающего тока. Если вспомнить обычные резисторы, то в них у нас сопротивление зависело только от самого резистора, материала, формы и всего такого прочего, но не зависело от частоты (разумеется, мы говорим сейчас про идеальные резисторы, без всяких паразитных параметров). Здесь все по-другому. Один и тот же конденсатор на разной частоте будет иметь разное сопротивление и через него будет течь ток разной амплитуды при одной и той же амплитуде напряжения.

Что еще мы можем сказать, глядя на эту формулу? Например, то, что чем больше частота сигнала, тем меньше для него сопротивление конденсатора. И чем больше емкость конденсатора, тем меньше его сопротивление переменному току.

По аналогии с резисторами, сопротивление конденсаторов измеряется все так же в Омах. Однако всегда следует помнить, что это немного другое сопротивление, его называют реактивным. И другое оно в первую очередь из-за того самого пресловутого j в знаменателе, то есть из-за сдвига фазы. У «обычных» (которые называют активными) Омов такого сдвига нет, там напряжение четко совпадает по фазе с током. Давайте построим график зависимости сопротивления конденсатора от частоты. Для определенности емкость конденсатора возьмем фиксированной, скажем, 1 мкФ. График представлен на рисунке 2.

Рисунок 2 (кликабельно) – Зависимость сопротивления конденсатора от частоты

На рисунке 2 мы видим, что сопротивление конденсатора переменному току убывает по закону гиперболы.

При стремлении частоты к нулю (то есть фактически при стремлении переменного току к постоянному) сопротивление конденсатора стремится к бесконечности. Это и логично: мы все помним, что для постоянного тока конденсатор фактически представляет собой разрыв цепи. На практике оно, конечно, не бесконечно, а ограничено сопротивлением утечки конденсатора. Тем не менее, оно все равно очень велико и часто его и считают бесконечно большим.

При стремлении частоты к бесконечности, сопротивление конденсатора стремится к нулю. Это все в теории, конечно. На практике реальный конденсатор обладает рядом паразитных параметров (в частности, паразитная индуктивности и сопротивление утечки), из-за чего сопротивление уменьшается только лишь до некоторой определенной частоты, а потом начинает наоборот расти. Но об этом более подробно в другой раз.

Есть еще один вопрос, который хотелось бы обговорить, прежде чем начинать рассмотрение примеров. Зачем вообще писать букву j в знаменателе сопротивления? Не достаточно ли просто всегда помнить про сдвиг фаз, а в записи использовать числа без этой мнимой единицы? Оказывается, нет. Представим себе цепь, где одновременно присутствуют резистор и конденсатор. Скажем, они соединены последовательно. И вот тут-то как раз мнимая единичка рядом с емкостью не позволит просто так взять и сложить активное и реактивное сопротивление в одно действительное число. Общее сопротивление такой цепочки будет комплексным, причем состоящим как из действительной части, так и из мнимой. Действительная часть будет обусловлена резистором (активными сопротивлением), а мнимая – емкостью (реактивным сопротивлением). Впрочем, это все тема для другой статьи, сейчас не будем в это углубляться. Давайте лучше перейдем к примерам.

Пусть у нас есть конденсатор емкостью, скажем C=1 мкФ. Требуется определить его сопротивление на частоте f1=50 Гц и на частоте f2=1 кГц. Кроме того, следует определить амплитуду тока с учетом того, что амплитуда приложенного к конденсатору напряжения равна Um=50 В. Ну и построить графики напряжения и тока.

Собственно, задачка эта элементарная. Подставляем циферки в формулу для сопротивления и получаем для частоты f1=50 Гц сопротивление, равное

А для частоты f2=1 кГц сопротивление будет

По закону Ома находим величину амплитуды тока для частоты f1=50 Гц

Аналогично для второй частоты f2=1 кГц

Теперь мы легко можем записать законы изменения тока и напряжения, а также построить графики для этих двух случаев. Полагаем, что напряжение у нас изменяется по закону синуса для первой частоты f1=50 Гц следующим образом

А для второй частоты f2=1 кГц вот так

Дальше мы помним, что ток в конденсаторе опережает напряжение на . Поэтому с учетом этого можем записать закон изменения тока через конденсаторы для первой частоты f1=50 Гц

и для частоты f2=1 кГц

Графики тока и напряжения для частоты f1=50 Гц представлены на рисунке 3

Рисунок 3 (кликабельно) – Напряжение на конденсаторе и ток через конденсаторе, f1=50 Гц

Графики тока и напряжения для частоты f2=1 кГц представлены на рисунке 4

Рисунок 4 (кликабельно) – Напряжение на конденсаторе и ток через конденсаторе, f2=1 кГц

Итак, господа, мы сегодня познакомились с таким понятием, как сопротивление конденсатора переменному току, научились его считать и закрепили полученные знания парочкой примеров. На сегодня все. Спасибо что прочитали, всем огромной удачи и пока!

 

Вступайте в нашу группу Вконтакте

Вопросы и предложения админу: This email address is being protected from spambots. You need JavaScript enabled to view it.


Как рассчитать реактивное сопротивление конденсатора для прямоугольной волны

Точно так же, как рассчитать реактивное сопротивление для конденсатора, когда через него проходит прямоугольная волна. Какая формула?

Нет емкостного сопротивления, связанного с прямоугольной волной. Само понятие реактивного сопротивления зависит от контекста синусоидального возбуждения.

Когда мы решаем схемы переменного тока в векторной области, считается само собой разумеющимся, что схема находится в синусоидальном устойчивом состоянии, то есть все источники имеют синусоидальную форму с одинаковой частотой и все переходные процессы затухают.

Этот факт таков: нельзя осмысленно суммировать фазоры или реактивные сопротивления для синусоид разных частот .

Теперь, это не значит, что вы не можете применить концепцию реактивного сопротивления, чтобы найти напряжение на конденсаторе для тока прямоугольной формы.

Поскольку (идеальные) конденсаторы являются линейными , мы можем разложить прямоугольную волну на синусоидальные компоненты, найти соответствующее синусоидальное напряжение для каждого компонента, а затем суммировать с компонентами напряжения, чтобы найти общее напряжение.

Напомним фундаментальное соотношение векторной области для напряжения и тока конденсатора:

В ⃗ с = 1 J ω C я ⃗ с В → с знак равно 1 J ω С я → с

где ω ω угловая частота связанной синусоиды.

Теперь пусть

я С ( т ) = а 1 соз ( ω т + ϕ 1 ) + а 2 соз ( 2 ω t + ϕ 2 ) + а 3 соз ( 3 ω т + ϕ 3 ) + . , , я С ( T ) знак равно 1 соз ⁡ ( ω T + φ 1 ) + 2 соз ⁡ ( 2 ω T + φ 2 ) + 3 соз ⁡ ( 3 ω T + φ 3 ) + , , ,

Для каждого синусоидального компонента существует связанный вектор. Например, для первого компонента связанный вектор

я ⃗ с 1 = а 1 е j ϕ 1 я → с 1 знак равно 1 е J φ 1

таким образом

В ⃗ с 1 = а 1 е j ϕ 1 J ω C В → с 1 знак равно 1 е J φ 1 J ω С

и что

v С 1 ( т ) = а 1 ω C соз ( ω т + ϕ 1 — π 2 ) v С 1 ( T ) знак равно 1 ω С соз ⁡ ( ω T + φ 1 — π 2 )

Повторите для каждого члена в серии, а затем сложите, чтобы найти общее напряжение конденсатора.

Обратите внимание, что мы не определили реактивное сопротивление для всей текущей формы волны, и мы не можем определить такую ​​вещь. Вместо этого мы

(1) найдено реактивное сопротивление для каждого синусоидального компонента

(2) преобразовал каждое результирующее векторное напряжение обратно во временную область

(3) суммируются отдельные компоненты напряжения во временной области

Емкостное сопротивление конденсатора формула

Реактивное сопротивление – электрическое сопротивление переменному току, обусловленное передачей энергии магнитным полем в индуктивностях или электрическим полем в конденсаторах.

Элементы, обладающие реактивным сопротивлением, называют реактивными.

Реактивное сопротивление катушки индуктивности.

При протекании переменного тока I в катушке, магнитное поле создаёт в её витках ЭДС, которая препятствует изменению тока.
При увеличении тока, ЭДС отрицательна и препятствует нарастанию тока, при уменьшении — положительна и препятствует его убыванию, оказывая таким образом сопротивление изменению тока на протяжении всего периода.

В результате созданного противодействия, на выводах катушки индуктивности в противофазе формируется напряжение U, подавляющее ЭДС, равное ей по амплитуде и противоположное по знаку.

При прохождении тока через нуль, амплитуда ЭДС достигает максимального значения, что образует расхождение во времени тока и напряжения в 1/4 периода.

Если приложить к выводам катушки индуктивности напряжение U, ток не может начаться мгновенно по причине противодействия ЭДС, равного -U, поэтому ток в индуктивности всегда будет отставать от напряжения на угол 90°. Сдвиг при отстающем токе называют положительным.

Запишем выражение мгновенного значения напряжения u исходя из ЭДС (ε), которая пропорциональна индуктивности L и скорости изменения тока: u = -ε = L(di/dt).
Отсюда выразим синусоидальный ток .

Интегралом функции sin(t) будет -соs(t), либо равная ей функция sin(t-π/2).
Дифференциал dt функции sin(ωt) выйдет из под знака интеграла множителем 1.
В результате получим выражение мгновенного значения тока со сдвигом от функции напряжения на угол π/2 (90°).
Для среднеквадратичных значений U и I в таком случае можно записать .

В итоге имеем зависимость синусоидального тока от напряжения согласно Закону Ома, где в знаменателе вместо R выражение ωL, которое и является реактивным сопротивлением:

Реактивное сопротивлениие индуктивностей называют индуктивным.

Реактивное сопротивление конденсатора.

Электрический ток в конденсаторе представляет собой часть или совокупность процессов его заряда и разряда – накопления и отдачи энергии электрическим полем между его обкладками.

В цепи переменного тока, конденсатор будет заряжаться до определённого максимального значения, пока ток не сменит направление на противоположное. Следовательно, в моменты амплитудного значения напряжения на конденсаторе, ток в нём будет равен нулю. Таким образом, напряжение на конденсаторе и ток всегда будут иметь расхождение во времени в четверть периода.

В результате ток в цепи будет ограничен падением напряжения на конденсаторе, что создаёт реактивное сопротивление переменному току, обратно-пропорциональное скорости изменения тока (частоте) и ёмкости конденсатора.

Если приложить к конденсатору напряжение U, мгновенно начнётся ток от максимального значения, далее уменьшаясь до нуля. В это время напряжение на его выводах будет расти от нуля до максимума. Следовательно, напряжение на обкладках конденсатора по фазе отстаёт от тока на угол 90 °. Такой сдвиг фаз называют отрицательным.

Ток в конденсаторе является производной функцией его заряда i = dQ/dt = C(du/dt).
Производной от sin(t) будет cos(t) либо равная ей функция sin(t+π/2).
Тогда для синусоидального напряжения u = U ampsin(ωt) запишем выражение мгновенного значения тока следующим образом:

Отсюда выразим соотношение среднеквадратичных значений .

Закон Ома подсказывает, что 1/ωC есть не что иное, как реактивное сопротивление для синусоидального тока:

Реактивное сопротивление конденсатора в технической литературе часто называют ёмкостным. Может применяться, например, в организации ёмкостных делителей в цепях переменного тока.

Онлайн-калькулятор расчёта реактивного сопротивления

Необходимо вписать значения и кликнуть мышкой в таблице.
При переключении множителей автоматически происходит пересчёт результата.

Расчитать ёмкость или индуктивность для реактивного сопротивления:

Похожие страницы с расчётами:

Замечания и предложения принимаются и приветствуются!

Одним из основных устройств в электронике и электротехнике является конденсатор. После замыкания электрической цепи начинается зарядка, после чего он сразу же становится источником тока и напряжения, в нем возникает электродвижущая сила – ЭДС. Одно из основных свойств конденсатора очень точно отражает формула емкостного сопротивления. Данное явление возникает в результате противодействия ЭДС, направленного против источника тока, используемого для зарядки. Источник тока может преодолеть емкостное сопротивление лишь путем существенных затрат его собственной энергии, которая становится энергией электрического поля конденсатора.

При разрядке устройства вся эта энергия возвращается обратно в цепь, превращаясь в энергию электрического тока. Поэтому емкостное сопротивление можно отнести к реактивному, не вызывающему безвозвратных энергетических потерь. Зарядка конденсатора происходит до того уровня напряжения, которое отдается источником питания.

Емкостное сопротивление конденсатора

Конденсаторы относятся к наиболее распространенным элементам, используемым в различных электронных схемах. Они разделяются на типы, обладающие характерными особенностями, параметрами и индивидуальными свойствами. Простейший конденсатор состоит из двух металлических пластин – электродов, разделенных слоем диэлектрика. На каждом из них имеется собственный вывод, через который осуществляется подключение к электрической цепи.

Существуют качества, присущие только конденсаторам. Например, они совершенно не пропускают через себя постоянный ток, хотя и заряжаются от него. После полной зарядки емкости, течение тока полностью прекращается, а внутреннее сопротивление устройства принимает бесконечно высокое значение.

Совершенно по-другому на конденсатор воздействует переменный ток, вполне свободно протекающий через емкость. Подобное состояние объясняется постоянными процессами зарядки-разрядки элемента. В этом случае действует не только активное сопротивление проводников, но и емкостное сопротивление самого конденсатора, возникающее как раз в результате его постоянной зарядки и разрядки.

Электрические параметры и свойства конденсаторов могут отличаться, в зависимости от различных факторов. В первую очередь они зависят от размеров и формы изделия, а также от типа диэлектрика. В разных типах устройств диэлектриком может служить бумага, воздух, пластик, стекло, слюда, керамика и другие материалы. В электролитических конденсаторах используются алюминий-электролит и тантал-электролит, что обеспечивает им повышенную емкость.

Названия других элементов определяются материалами обычных диэлектриков. Поэтому они относятся к категории бумажных, керамических, стеклянных и т.д. Каждый из них, в соответствии с характеристиками и особенностями, применяется в конкретных электронных схемах, с разными параметрами электротока.

В связи с этим, применение керамических конденсаторов необходимо в тех цепях, где требуется фильтрация высокочастотных помех. Электролитические устройства, наоборот, фильтруют помехи при низких частотах. Если же соединить параллельно оба типа конденсаторов, получится универсальный фильтр, широко применяемый во всех схемах. Несмотря на то, что их емкость является фиксированной величиной, существуют устройства с переменной емкостью, которая достигается путем регулировок за счет изменение взаимного перекрытия пластин. Типичным примером служат конденсаторы для подстройки, используемые при регулировке радиоэлектронной аппаратуры.

Емкостное сопротивление в цепи переменного тока

При включении конденсатора в цепь постоянного тока, на протяжении короткого периода времени будет наблюдаться течение по цепи зарядного тока. По окончании зарядки, когда напряжение конденсатора будет соответствовать напряжению источника тока, кратковременное течение тока в цепи прекратится. Таким образом, полностью заряженный конденсатор при постоянном токе будет своеобразным разрывом цепи или сопротивлением с бесконечно большим значением. При переменном токе конденсатор будет вести себя совершенно иначе. Его зарядка в такой цепи будет осуществляться поочередно в разных направлениях. Течение переменного тока в цепи в это время не прерывается.

Более подробное рассмотрение этого процесса указывает на нулевое значение напряжения в конденсаторе в момент его включения. После поступления к нему переменного напряжения сети начнется зарядка. В это время сетевое напряжение будет возрастать на протяжении первой четверти периода. По мере того как на обкладках накапливаются заряды, происходит увеличение напряжения самого конденсатора. После того как сетевое напряжение в конце первой четверти периода станет максимальным, зарядка прекращается и значение тока в цепи станет равным нулю.

Существует формула для определения тока в цепи конденсатора: I = ∆q/∆t, где q является количеством электричества, протекающим по цепи в течение промежутка времени t. В соответствии с законами электростатики, количество электричества в устройстве составит: q = C x Uc = C x U. В этой формуле С будет емкостью конденсатора, U – напряжением сети, Uc – напряжением на обкладках элемента. В окончательном виде формула тока в цепи будет выглядеть следующим образом: i = C x (∆Uc/∆t) = C x (∆U/∆t).

При наступлении второй четверти периода произойдет уменьшение сетевого напряжения и начнется разрядка конденсатора. Ток в цепи изменит свое направление и будет течь в обратную сторону. В следующей половине периода направление сетевого напряжения изменится, наступит перезарядка элемента, а потом он вновь начнет разряжаться. Ток, присутствующий в цепи с конденсаторной емкостью, будет опережать по фазе напряжение на обкладках на 90 градусов.

Установлено что изменения тока конденсатора происходят со скоростью, находящейся в пропорциональной зависимости с угловой частотой ω. Поэтому в соответствии с уже известной формулой тока в цепи i = C x (∆U/∆t), аналогично получается, что действующее значение тока также будет представлять собой пропорцию между скоростью изменения напряжения и угловой частотой ω: I = 2π x f x C x U.

Далее уже совсем несложно установить значение емкостного сопротивления или реактивного сопротивления емкости: xc = 1/2π x f x C = 1/ ω x C. Данный параметр вычисляется, когда конденсаторная емкость включается в цепь переменного тока. Поэтому в соответствии с законом Ома в цепи переменного тока с включенным конденсатором, значение силы тока будет следующим: I = U/xc, а напряжение на обкладках составит: Uc = Ic x xc.

Часть сетевого напряжения, приходящаяся на конденсатор, получила название емкостного падения напряжения. Она известна также, как реактивная слагающая напряжения, обозначаемая символом Uc. Величина емкостного сопротивления хс, так же, как и значение индуктивного сопротивления xi напрямую связана с частотой переменного тока.

Электросопротивление — это параметр в электротехнике, характеризующий возможность вещества препятствовать прохождению электричества. В зависимости от качеств материала, электросопротивляемость может уменьшаться до крайне маленьких величин (микромилиОмы — у проводников, металлов) или повышаться до огромных значений (ГигаОмы — изоляторов, диэлектриков). Величина противоположная сопротивлению — проводимость.

Что такое

Цепь, по которой протекает непостоянный ток, обладает полным сопротивлением. Вычисляется оно по сумме активного и реактивного сопротивлений, возведенных в квадрат.

Графическое изображение этой формулы представляет собой треугольник. Его катеты представлены активным и реактивным сопротивлениями, а гипотенуза полным электросопротивлением.

Емкостное электросопротивление (Xc) является одним из видов реактивного сопротивления. Этот показатель характеризует противодействие электроемкости в цепи электротоку с переменными параметрами. Преобразование электроэнергии в тепловую в момент протекания электричества сквозь емкость не возникает (свойство реактивного сопротивления). Вместо этого осуществляется передача энергии электрического тока электрическому полю и обратно. Потерь энергии при таком обмене не происходит.

Емкостное сопротивление конденсатора можно сравнить с кастрюлей, наполняемой жидкостью, при полном заполнении ее объема она переворачивается, выливая содержимое, а затем наполняется заново. После достижения максимального заряда конденсатора происходит разрядка, затем он заряжается вновь.

Дополнительная информация: Конденсатор цепи способен накопить лишь ограниченную величину заряда до перемены полярности напряжения. По данной причине непостоянный ток не падает до нуля, важное отличие от постоянного электричества. Низкие значения частоты тока соответствуют низким показателям заряда, накопленного конденсатором, низким значениям противодействия электричеству, что придает реактивные свойства.

По сути, Xc — это противостояние электродвижущей силы конденсатора, уровню его заряда.

От чего зависит сопротивление конденсаторов цепей переменного тока

Показатели его, зависят не только от емкостных характеристик последнего, но и от частотной характеристики электротока, протекающего по цепи. Когда речь идет о сопротивлении резистора, то говорится о параметрах самого резистора, например, материале, форме, но полностью отсутствует взаимосвязь сопротивления его и показателей частоты электричества цепи (речь идет об идеальном резисторе, паразитные параметры которому не характерны). Когда речь идет об устройстве накопления энергии и заряда электрического поля — все иначе. Конденсатор одной и той же емкости при разных частотах тока обладает неодинаковым уровнем сопротивления. Амплитуда протекающего через него электричества при постоянной амплитуде напряжения обладает разной величиной.

Рассматривая эту формулу сопротивления конденсатора в цепи переменного тока, к каким выводам можно прийти? При повышении частотных показателей сигнала, электросопротивляемость конденсатора снижается.

При повышении емкостных характеристик устройства для накопления заряда и энергии электрического поля Xc переменного электричества, проходящего сквозь него, будет стремиться вниз.

Момент приближения значений частоты к нулевым отметкам на оси (когда переменный электроток становится похож своими параметрами на постоянный), сопровождается возрастанием Xc конденсатора до беспредельных величин. Это действительно так: известно, что конденсатор сети постоянного тока является фактически разрывом цепи. Реальная электросопротивляемость, естественно, не бесконечна, ее ограничивает уровень конденсаторной утечки. Но величины его остаются на высоком уровне, который невозможно не учитывать.

При возрастании цифр частоты до уровня бесконечных значений, емкостное сопротивление электроконденсатора стремится к нулевым отметкам. Такое характеризует идеальные модели. В реальных условиях конденсатор имеет неприятные характеристики (такие как индуктивность и сопротивления утечек), поэтому снижение емкостного сопротивления происходит до определенных значений, после которых оно возрастает.

Обратите внимание! При подключении конденсатора к цепочке электричества с переменными параметрами, его мощность не тратится, потому что фазовые характеристики напряжения и силы тока сдвинуты на 90° в отношении друг друга. В одну четверть периода происходит зарядка электроконденсатора (энергия запасается в его электрополе), в следующее время происходит его разрядка, энергия поступает обратно в цепочку. Его электросопротивляемость является безваттной, реактивной.

Причины ёмкостного сопротивления

Причиной возникновения сопротивления емкостного считается уровень напряжения, возникающий на конденсаторе в процессе его заряда. Вектор его действия встречен вектору напряжения источника электричества, потому создает помеху воспроизведению электротока этим источником.

Как рассчитать Xc

Сила тока цепи с постоянными показателями напряжения в момент работы электроконденсатора равно 0. Ее значения в цепи с переменным напряжением после подключения конденсатора I ? 0. В итоге, цепочке с непостоянным напряжением конденсатор придает Xc меньшее, чем цепочке с неизменным показателем напряжения.

Получается, что изменения напряжения отличаются по фазе от изменений тока на π/2.

По закону, сформулированному Омом, показатели силы электротока находятся в прямой пропорциональной зависимости от величины напряжения цепи. Формула вычисления наибольших величин напряженности и силы тока:

f — показатель частоты непостоянного тока, измеряется в герцах;

ω — показатель угловой частоты тока;

С — размер конденсатора в фарадах.

Важно! Xc не выступает параметром проводника, оно находится в зависимости от такой характеристики электроцепи, как частота электротока.

Повышение значений данной величины вызывает рост пропускающей способности конденсатора (предел его сопротивления току непостоянному понижается).

Представим, к цепи подключен конденсатор, емкостью 1 мкФ. Необходимо вычислить, уровень емкостного сопротивления при величине частоты 50 Гц и как изменится емкостное сопротивление цепи переменного тока при частоте 1 кГц. Амплитуда напряжения, подведенного к конденсатору, составляет 50 В.

После введения данных в формулу, определяющую Xc, и получаются значения:

Емкостное сопротивление приравнивается к соотношению отклонений колебаний напряжения зажимов электрической цепочки с емкостными параметрами (с небольшими индуктивным и активным сопротивлениями) к колебаниям электротока цепочки. Она равнозначна электроконденсатору.

В чем измеряется емкостное электросопротивление

R представлено отношением напряжения к силе тока замкнутой электрической цепи, по закону Ома. Единицы измерения — Ом. Xc, как его разновидность, тоже измеряется в Омах.

Конденсаторы применяются при изготовлении фильтров. При параллельном присоединении к цепи, он способен задерживать высокие частоты, при последовательном удаляет низкие. Также они используются с целью отсечения переменной части от постоянной. Он незаменим в радиотехнике, при производстве датчиков приближения, для контроля процессов производства. Технологии, обладающие выше описанными свойствами, используются во всех областях промышленности.

Реактивное сопротивление

                                     

2. Индуктивное сопротивление

Индуктивное реактивное сопротивление — это свойство, проявляемое индуктивностью, и индуктивное реактивное сопротивление существует благодаря тому, что электрический ток создаёт вокруг него магнитное поле. В контексте цепи переменного тока хотя эта концепция применяется при любом изменении тока, это магнитное поле постоянно изменяется в результате изменения тока, который меняется во времени. Именно это изменение магнитного поля создаёт другой электрический ток в том же проводе противо-ЭДС, в направлении, противоположном потоку тока, изначально ответственного за создание магнитного поля. Это явление известно как закон Ленца. Следовательно, индуктивное сопротивление — это противодействие изменению тока через элемент.

Для идеальной катушки индуктивности в цепи переменного тока сдерживающее влияние на изменение протекания тока приводит к задержке или сдвигу фаз переменного тока относительно переменного напряжения. В частности, идеальная индуктивность без сопротивления вызовет отставание тока от напряжения на четверть цикла или на 90°.

В электроэнергетических системах индуктивное реактивное сопротивление и ёмкостное реактивное сопротивление, однако индуктивное реактивное сопротивление более распространено может ограничивать пропускную способность линии электропередач переменного тока, поскольку мощность не передаётся полностью, когда напряжение и ток находятся в противофазе подробно описано выше. То есть ток будет течь для противофазной системы, однако реальная мощность в определённые моменты времени не будет передаваться, потому что будут моменты, в течение которых мгновенный ток будет положительным, а мгновенное напряжение отрицательным, или наоборот, подразумевая отрицательную мощность передачи. Следовательно, реальная работа не выполняется, когда передача энергии является «отрицательной». Однако ток всё ещё течёт, даже когда система находится в противофазе, что приводит к нагреву линий электропередачи из-за протекания тока. Следовательно, линии электропередачи могут только сильно нагреваться иначе они физически сильно прогибаются из-за тепла, расширяющего металлические линии электропередачи, поэтому операторы линий электропередачи имеют «потолок» в отношении величины тока, который может протекать через данную линию, и чрезмерное индуктивное сопротивление ограничивает мощность линии. Поставщики электроэнергии используют конденсаторы для сдвига фазы и минимизации потерь в зависимости от схемы использования.

Индуктивное реактивное сопротивление X L {\displaystyle \scriptstyle {X_{L}}} пропорционально частоте синусоидального сигнала f {\displaystyle \scriptstyle {f}} и индуктивности L {\displaystyle \scriptstyle {L}}, которая зависит от геометрических размеров и формы индуктивности.

X L = ω L = 2 π f L {\displaystyle X_{L}=\omega L=2\pi fL}

Средний ток, протекающий через индуктивность L {\displaystyle \scriptstyle {L}} последовательно с синусоидальным источником переменного напряжения среднеквадратичной амплитуды A {\displaystyle \scriptstyle {A}} и частоты f {\displaystyle \scriptstyle {f}} равен:

I L = A ω L = A 2 π f L {\displaystyle I_{L}={A \over \omega L}={A \over 2\pi fL}}.

Поскольку прямоугольная волна источник прямоугольного сигнала имеет несколько амплитуд на синусоидальных гармониках согласно теореме Фурье, средний ток, протекающий через индуктивность L {\displaystyle \scriptstyle {L}}, включенную последовательно с прямоугольным источником переменного напряжения среднеквадратичной амплитуды A {\displaystyle \scriptstyle {A}} и частоты f {\displaystyle \scriptstyle {f}}, равен:

I L = A π 2 8 ω L = A π 16 f L {\displaystyle I_{L}={A\pi ^{2} \over 8\omega L}={A\pi \over 16fL}}

создавая иллюзию как если бы реактивное сопротивление прямоугольной волны на 19 % меньше X L = 16 π f L {\displaystyle X_{L}={16 \over \pi }fL}, чем реактивное сопротивление синусоидального сигнала с той же частотой:

Любой проводник конечных размеров имеет индуктивность; индуктивность обычно делается из электромагнитных катушек, состоящих из множества витков провода. Согласно закону электромагнитной индукции Фарадея возникает противоэдс E {\displaystyle \scriptstyle {\mathcal {E}}} ток, противоположный напряжению в проводнике из-за скорости изменения плотности магнитного потока B {\displaystyle \scriptstyle {B}} через токовую петлю.

E = − d Φ B d t {\displaystyle {\mathcal {E}}=-

Противо-ЭДС — это источник противодействия току. Постоянный ток имеет нулевую скорость изменения и рассматривает катушку индуктивности как обычный проводник так как она сделано из материала с низким удельным сопротивлением. Переменный ток имеет усреднённую по времени скорость изменения, которая пропорциональна частоте, что вызывает увеличение индуктивного сопротивления с частотой.

Емкостное сопротивление конденсатора формула. Реактивное сопротивление конденсатора

О заряде конденсатора.

Замкнем цепь. В цепи пойдет ток заряда конденсатора. Это значит что с левой обкладки конденсатора часть электронов уйдет в провод, а из провода на правую обкладку зайдет такое же количество электронов. Обе обкладки будут заряжены разноименными зарядами одинаковой величины.

Между обкладками в диэлектрике будет электрическое поле.

А теперь разомкнем цепь. Конденсатор останется заряженным. Закоротим куском провода его обкладки. Конденсатор мгновенно разрядится. Это значит что с правой обкладки уйдет в провод избыток электронов, а из провода на левую обкладку войдет недостаток электронов. На обоих обкладках электронов будет одинаково, конденсатор разрядится.

До какого напряжения заряжается конденсатор?

Он заряжается до такого напряжения, которое к нему приложено с источника питания.

Сопротивление конденсатора.


Замкнем цепь. Конденсатор начал заряжаться и сразу стал источником тока, напряжения, Э. Д. С.. На рисунке видно что Э. Д. С. конденсатора направлена против заряжающего его источника тока.

Противодействие электродвижущей силы заряжаемого конденсатора заряду этого конденсатора называется емкостным сопротивлением.

Вся энергия затрачиваемая источником тока на преодоление емкостного сопротивления превращается в энергию электрического поля конденсатора. Когда конденсатор будет разряжаться вся энергия электрического поля вернется обратно в цепь в виде энергии электрического тока. Таким образом емкостное сопротивление является реактивным, т.е. не вызывающим безвозвратных потерь энергии.

Почему постоянный ток не проходит через конденсатор, а переменный ток проходит?

Включим цепь постоянного тока. Лампа вспыхнет и погаснет, почему? Потому что в цепи прошел ток заряда конденсатора. Как только конденсатор зарядится до напряжения батареи ток в цепи прекратится.

А теперь замкнем цепь переменного тока. В I четверти периода напряжение на генераторе возрастает от 0 до максимума. В цепи идет ток заряда конденсатора. Во II четверти периода напряжение на генераторе убывает до нуля. Конденсатор разряжается через генератор. После этого конденсатор вновь заряжается и разряжается. Таким образом в цепи идут токи заряда и разряда конденсатора. Лампочка будет гореть постоянно.

В цепи с конденсатором ток проходит во всей замкнутой цепи, в том числе и в диэлектрике конденсатора. В заряжающемся конденсаторе образуется электрическое поле которое поляризует диэлектрик. Поляризация это вращение электронов в атомах на вытянутых орбитах.

Одновременная поляризация огромного количества атомов образует ток, называемый током смещения. Таким образом в проводах идет ток и в диэлектрике причем одинаковой величины.

Емкостное сопротивление конденсатора определяется по формуле

Рассматривая график делаем вывод: ток в цепи с чисто емкостным сопротивлением опережает напряжение на 90 0 .

Возникает вопрос каким образом ток в цепи может опережать напряжение на генераторе? В цепи идет ток от двух источников тока поочередно, от генератора и от конденсатора. Когда напряжение на генераторе равно нулю ток в цепи максимален. Это ток разряда конденсатора.

О реальном конденсаторе

Реальный конденсатор имеет одновременно два сопротивления: активное и емкостное. Их следует считать включенными последовательно.

Напряжение приложенное генератором к активному сопротивлению и ток идущий по активному сопротивлению совпадают по фазе.

Напряжение приложенное генератором к емкостному сопротивлению и ток идущий по емкостному сопротивлению сдвинуты по фазе на 90 0 . Результирующее напряжение приложенное генератором к конденсатору можно определить по правилу параллелограмма.

На активном сопротивлении напряжение U акт и ток I совпадают по фазе. На емкостном сопротивлении напряжение U c отстает от тока I на 90 0 . Результирующее напряжение приложенное генератором к конденсатору определяется по правилу параллелограмма. Это результирующее напряжение отстает от тока I на какой то угол φ всегда меньший 90 0 .

Определение результирующего сопротивления конденсатора

Результирующее сопротивление конденсатора нельзя находить суммируя величины его активного и емкостного сопротивлений. Это делается по формуле

При переменном напряжении на реальном конденсаторе кроме тока смещения имеются небольшие токи проводимости, через толщу диэлектрика (объемный ток) и по поверхности (поверхностный ток).Токи проводимости и поляризацию диэлектрика сопровождают потери энергии.

Таким образом, в реальном конденсаторе наряду с изменением энергии электрического поля (это характеризует реактивная мощность Q ) из-за несовершенства диэлектрика идет необратимый процесс преобразования электрической энергии в тепло, скорость которого выражается активной мощностью Р . Поэтому в схеме замещения реальный конденсатор должен быть представлен активным и реактивным элементами.

Деление реального конденсатора на два элемента — это расчетный прием, так как конструктивно их выделить нельзя. Однако такую же схему замещения имеет реальная цепь из двух элементов, один из которых характеризуется только активной мощностью Р (Q = 0), другой — реактивной (емкостной) мощностью Q(P = 0).

Схема замещения конденсатора с параллельным соединением элементов

Реальный конденсатор (с потерями) можно представить эквивалентной схемой параллельного соединения активной G и емкостной B с проводимостей (рис. 13.15), причем активная проводимость определяется мощностью потерь в конденсаторе G = Р/U c 2 , а емкость — конструкцией конденсатора. Предположим, что проводимости G и В с для такой цепи известны, а напряжение имеет уравнение

u = Umsinωt .

Требуется определить токи в цепи и мощность. Исследование цепи с активным сопротивлением и цепи с емкостью показало, что при синусоидальном напряжении токи в них так же синусоидальны. При параллельном соединении ветвей G и В с, согласно первому закону Кирхгофа, общий ток i равен сумме токов в ветвях с активной и емкостной проводимостями:

i = i G + i c , (13.30)

Учитывая, что ток i G совпадает по фазе с напряжением, а ток i c опережает напряжение на четверть периода, уравнение общего тока можно записать в следующем виде:


Векторная диаграмма токов в цепи с конденсатором

Для определения действующей величины общего тока I методом векторного сложения построим векторную диаграмму согласно уравнению

I = I G + I C

Действующие величины составляющих тока:

I G = GU (13.31)

I C = B C U (13.32)

Первым на векторной диаграмме изображается вектор напряжения U (рис. 13.16, а), его направление совпадает с положительным направлением оси, от которой отсчитываются фазовые углы (начальная фаза напряжения φ a =0). Вектор I G совпадает по направлению с вектором U, а вектор I C направлен перпендикулярно вектору U с положительным углом. Из векторной диаграммы видно, что вектор общего напряжения отстает от вектора общего тока на угол φ , величина которого больше нуля, но меньше 90º. Вектор I является гипотенузой прямоугольного треугольника, катеты которого — составляющие его векторы I G и I C:

При напряжении u = U m sinωt соответствии с векторной диаграммой уравнение тока

i = I m sin(ωt + φ )

Треугольник проводимостей для конденсатора

Стороны треугольников токов, выраженные в единицах тока, разделим на напряжение U. Получим подобный треугольник проводимостей (рис. 13.16, б), катетами которого являются активная G = I G /U и емкостная В с = I с /U проводимости, а гипотенузой — полная проводимость цепи Y = I/U . Из треугольника проводимостей

Связь между действующими величинами напряжения и тока выражается формулами

I = UY

U = I/Y (13.35)

Из треугольников токов и проводимостей определяют величины

cos φ = I G /I = G/Y; sinφ = I c /I = B c /Y; tgφ = I C /I G = B c /G. (13.36)

Мощность цепи с конденсатором

Выражение мгновенной мощности реального конденсатора

p = ui = U m sinωt * I m sin(ωt+φ)

совпадает с выражением мгновенной мощности катушки. Рассуждения, аналогичные тем, которые сделаны при рассмотрении графика мгновенной мощности (см. рис.13. 11), можно провести и для реального конденсатора на основе графика рис. 13.17. Величины активной, реактивной и полной мощностей выражаются теми же формулами, какие были получены для катушки [см. (13.19) — (13.22)]. Это нетрудно показать, если стороны треугольника токов, выраженные в единицах тока, умножить на напряжение U. В результате умножения получится подобный треугольник мощностей (рис. 13.16, в), катетами которого являются мощности; активная

P = UI G = UIcosφ

реактивная

Q = UI C = UIsinφ

полная

Схема замещения конденсатора с последовательным соединением элементов

Реальный конденсатор, так же как и , на расчетной схеме может быть представлен последовательным соединением двух участков: с активным R и емкостным Х с сопротивлениями. На рис. 13.18, а такая схема показана в сравнении со схемой параллельного соединения активной и емкостной проводимостей (рис.13. 18,6). Все выводы и формулы, полученные для катушки, остаются в силе и для конденсатора при условии замены индуктивного сопротивления емкостным. Конденсаторы, применяемые на практике, имеют относительно малые потери энергии. Поэтому в схемах замещения они представлены чаще всего только реактивной частью, т. е. емкостью С Участки цепи, где последовательно соединены отдельные элементы — резистор R и конденсатор С, имеют такую схему замещения, как показано на рис. 13.18, а. Если вам интересно прочитайте которые применяются в промышленности.

Положим теперь, что участок цепи содержит конденсатор емкости C , причем сопротивлением и индуктивностью участка можно пренебречь, и посмотрим, по какому закону будет изменяться напряжение на концах участка в этом случае. Обозначим напряжение между точками а и b через u и будем считать заряд конденсатора q и силу тока i положительными, если они соответствуют рис.4. Тогда

и, следовательно,

Если сила тока в цепи изменяется по закону

то заряд конденсатора равен

.

Постоянная интегрирования q 0 здесь обозначает произвольный постоянный заряд конденсатора, не связанный с колебаниями тока, и поэтому мы положим . Следовательно,

. (2)

Сравнивая (1) и (2), мы видим, что при синусоидальных колебаниях тока в цепи напряжение на конденсаторе изменяется также по закону косинуса. Однако колебания напряжения на конденсаторе отстают по фазе от колебаний тока на p/2. Изменения тока и напряжения во времени изображены графически на рис.5. Полученный результат имеет простой физический смысл. Напряжение на конденсаторе в какой-либо момент времени определяется существующим зарядом конденсатора. Но этот заряд был образован током, протекавшим предварительно в более ранней стадии колебаний. Поэтому и колебания напряжения запаздывают относительно колебаний тока.

Формула (2) показывает, что амплитуда напряжения на конденсаторе равна

Сравнивая это выражение с законом Ома для участка цепи с постоянным током (), мы видим, что величина

играет роль сопротивления участка цепи, она получила название емкостного сопротивления. Емкостное сопротивление зависит от частоты w, и при высоких частотах даже малые емкости могут представлять совсем небольшое сопротивление для переменного тока. Важно отметить, что емкостное сопротивление определяет связь между амплитудными, а не мгновенными значениями тока и напряжения.

Мгновенная мощность переменного тока

меняется со временем по синусоидальному закону с удвоенной частотой. В течение времени от 0 до T /4 мощность положительна, а в следующую четверть периода ток и напряжение имеют противоположные знаки и мощность становится отрицательной. Поскольку среднее значение за период колебаний величины равно нулю, то средняя мощность переменного тока на конденсаторе .

Ток в цепи с конденсатором может протекать лишь при изменении приложенного к ней напряжения, причем сила тока, протекающего по цепи при заряде и разряде конденсатора, будет тем больше, чем больше емкость конденсатора и чем быстрее происходят изменения ЭДС.
Конденсатор, включенный в цепь переменного тока, влияет на силу протекающего по цепи тока, т. е. ведет себя как сопротивление. Величина емкостного сопротивления тем меньше, чем больше емкость и чем выше частота переменного тока. И наоборот, сопротивление конденсатора переменному току увеличивается с уменьшением его емкости и понижением частоты.

где Xc — реактивное сопротивление конденсатора, f — частота, C — емкость.

Для расчета реактивного сопротивления конденсатора заполните предложенную форму:

Расчет ёмкости для реактивного сопротивления:

Расчёт ёмкости: C = 1 /(2πƒX C)
  • Похожие статьи
  • — Бестрансформаторные источники питания с гасящим конденсатором удобны своей простотой, имеют малые габариты и массу, но не всегда применимы из-за гальванической связи выходной цепи с сетью 220 В. В бестрансформаторном источнике питания к сети переменного напряжения подключены последовательно…
  • — Принципиальная электрическая схема цифрового широкодиапазонного измерителя емкости показана на рисунке. Принцип работы прибора – измерение длительности импульса автогенератора, в состав времязадающей цепи которого входит измеряемый конденсатор. Далее, формируется пачка импульсов образцовой частоты…
  • — Данная статья посвящена простому блоку со стабилизатором типа КРЕН. КРЕН это 3-х или 4-х выводные микросхемы, для примера взята 3-х выводная микросхема. Для стабилизированного напряжения (положительного) можно взять микросхему КРЕН5А, на +5В. Силовая часть (см. рис.1) примерно одинакова для…
  • — Габариты и масса высоковольтных трансформаторов из-за необходимости обеспечения электрической прочности становятся очень большими. Поэтому удобнее использовать в высоковольтных маломощных источниках питания умножители напряжения. Умножители напряжения создаются на базе схем выпрямления с емкостной…
  • — Приемник может быть перестроен в диапазоне 70…150 МГц без изменения номиналов подстроечных элементов. Реальная чувствительность приемника около 0,3 мкВ, напряжение питания 9 В. Следует заметить, что напряжение питания МС3362 — 2…7 В, а МС34119 2…12 В, поэтому МС3362 питается через…

Как найти сопротивление катушки формула

ЭКСПЕРИМЕНТ 18 Катушки индуктивности и переменный ток

После проведения данного эксперимента Вы сможете объяснить эффект индуктивности в схеме переменного тока и рассчитать значения индуктивности и реактивного сопротивления по результатам измерении.

* Катушка индуктивности 100 мГн

* Генератор функций / сигнал-генератор

Когда катушка индуктивности включается в цепь переменного тока, непрерывные изменения напряжения приводят к изменениям тока, которые в свою очередь генерируют то возрастающее, то убывающее магнитное поле. Это магнитное поле индуцирует встречное напряжение в катушке индуктивности, и оно противодействует изменениям тока. В результате имеет место непрерывное противодействие протеканию тока. Это противодействие называется индуктивным сопротивлением (XL).

формула индуктивного сопротивления

Индуктивное сопротивление катушки или дросселя зависит от частоты приложенного переменного напряжения (f) и значения индуктивности (L) в генри. Для вычисления индуктивного сопротивления, выражаемого в омах, служит простая формула:

Индуктивное сопротивление прямо пропорционально частоте и индуктивности. Если известно индуктивное сопротивление, путем преобразования основной формулы может быть найдена или частота, или индуктивность, как показано ниже:

формула полного сопротивления

Вспомните, что чистых индуктивностей нет, поскольку катушки индуктивности сделаны с использованием проволоки, которая имеет сопротивление. Полное сопротивление, оказываемое катушкой индуктивности переменному току, представляет собой, следовательно, комбинацию индуктивного сопротивления и обычного (активного) сопротивления. Это комбинированное противодействие известно как полное сопротивление (или импеданс). Полное сопротивление может быть вычислено при помощи формулы:

Вспомните, что индуктивность приводит к запаздыванию тока относительно напряжения. По

этой причине напряжения на катушке индуктивности и на резисторе сдвинуты по фазе на 90 градусов друг относительно друга. Это как раз и не позволяет нам просто сложить вместе индуктивное сопротивление и активное, сопротивление, чтобы получить величину импеданса.

Если известно полное сопротивление, а индуктивное сопротивление или активное сопротивление неизвестно, предыдущая формула может быть преобразована для их нахождения следующим образом:

Если известно полное сопротивление индуктивной схемы, Вы можете рассчитать ток в схеме, если Вы знаете приложенное напряжение. Это осуществляется применением закона Ома:

Естественно, эта формула также может быть преобразована для вычисления двух других переменных, если это потребуется:

В данном эксперименте Вы познакомитесь с эффектом индуктивности в схеме переменного тока.

1. Измерьте сопротивление обмотки катушки индуктивности при помощи мультиметра.

Сопротивление постоянному току =____ Ом

2. Присоедините катушку индуктивности 100 мГн к сигнал-генератору, формирующему напряжение размаха 4 Vpp с частотой 400 Гц.

3. Теперь измерьте фактическое значение тока первичной обмотки. Вспомните, что амперметр должен включаться последовательно со схемой для выполнения измерения. Подключите мультиметр для измерения переменного тока. Убедитесь, что генератор продолжает формировать 4 Vpp.

4. Используя информацию, которую Вы собрали

в предыдущих шагах, и формулы, приведенные в вводной части, рассчитайте полное сопротивление схемы.

5. Используя информацию, которую Вы собрали в предыдущих шагах, и формулы, приведенные в вводной части, рассчитайте индуктивность (L) катушки. L = _____ мГн

1. При увеличении частоты переменного тока, пропускаемого через катушку индуктивности, индуктивное сопротивление:

в) остается без изменения.

2. При уменьшении величины индуктивности в схеме индуктивное сопротивление:

в) остается без изменения.

3. При уменьшении сопротивления катушки индуктивности ее полное сопротивление:

в) остается без изменения.

4. Единицей измерения для величины индуктивного сопротивления является:

5. Катушка индуктивности имеет (активное) сопротивление 120 Ом. Когда к катушке прикладывается переменное напряжение 24 В с частотой 60Гц, протекает ток 111 мА. Значение индуктивности составляет приблизительно:

Реактивное сопротивление – электрическое сопротивление переменному току, обусловленное передачей энергии магнитным полем в индуктивностях или электрическим полем в конденсаторах.

Элементы, обладающие реактивным сопротивлением, называют реактивными.

Реактивное сопротивление катушки индуктивности.

При протекании переменного тока I в катушке, магнитное поле создаёт в её витках ЭДС, которая препятствует изменению тока.
При увеличении тока, ЭДС отрицательна и препятствует нарастанию тока, при уменьшении – положительна и препятствует его убыванию, оказывая таким образом сопротивление изменению тока на протяжении всего периода.

В результате созданного противодействия, на выводах катушки индуктивности в противофазе формируется напряжение U, подавляющее ЭДС, равное ей по амплитуде и противоположное по знаку.

При прохождении тока через нуль, амплитуда ЭДС достигает максимального значения, что образует расхождение во времени тока и напряжения в 1/4 периода.

Если приложить к выводам катушки индуктивности напряжение U, ток не может начаться мгновенно по причине противодействия ЭДС, равного -U, поэтому ток в индуктивности всегда будет отставать от напряжения на угол 90°. Сдвиг при отстающем токе называют положительным.

Запишем выражение мгновенного значения напряжения u исходя из ЭДС (ε), которая пропорциональна индуктивности L и скорости изменения тока: u = -ε = L(di/dt).
Отсюда выразим синусоидальный ток .

Интегралом функции sin(t) будет -соs(t), либо равная ей функция sin(t-π/2).
Дифференциал dt функции sin(ωt) выйдет из под знака интеграла множителем 1.
В результате получим выражение мгновенного значения тока со сдвигом от функции напряжения на угол π/2 (90°).
Для среднеквадратичных значений U и I в таком случае можно записать .

В итоге имеем зависимость синусоидального тока от напряжения согласно Закону Ома, где в знаменателе вместо R выражение ωL, которое и является реактивным сопротивлением:

Реактивное сопротивлениие индуктивностей называют индуктивным.

Реактивное сопротивление конденсатора.

Электрический ток в конденсаторе представляет собой часть или совокупность процессов его заряда и разряда – накопления и отдачи энергии электрическим полем между его обкладками.

В цепи переменного тока, конденсатор будет заряжаться до определённого максимального значения, пока ток не сменит направление на противоположное. Следовательно, в моменты амплитудного значения напряжения на конденсаторе, ток в нём будет равен нулю. Таким образом, напряжение на конденсаторе и ток всегда будут иметь расхождение во времени в четверть периода.

В результате ток в цепи будет ограничен падением напряжения на конденсаторе, что создаёт реактивное сопротивление переменному току, обратно-пропорциональное скорости изменения тока (частоте) и ёмкости конденсатора.

Если приложить к конденсатору напряжение U, мгновенно начнётся ток от максимального значения, далее уменьшаясь до нуля. В это время напряжение на его выводах будет расти от нуля до максимума. Следовательно, напряжение на обкладках конденсатора по фазе отстаёт от тока на угол 90 °. Такой сдвиг фаз называют отрицательным.

Ток в конденсаторе является производной функцией его заряда i = dQ/dt = C(du/dt).
Производной от sin(t) будет cos(t) либо равная ей функция sin(t+π/2).
Тогда для синусоидального напряжения u = U ampsin(ωt) запишем выражение мгновенного значения тока следующим образом:

Отсюда выразим соотношение среднеквадратичных значений .

Закон Ома подсказывает, что 1/ωC есть не что иное, как реактивное сопротивление для синусоидального тока:

Реактивное сопротивление конденсатора в технической литературе часто называют ёмкостным. Может применяться, например, в организации ёмкостных делителей в цепях переменного тока.

Онлайн-калькулятор расчёта реактивного сопротивления

Необходимо вписать значения и кликнуть мышкой в таблице.
При переключении множителей автоматически происходит пересчёт результата.

Расчитать ёмкость или индуктивность для реактивного сопротивления:

Похожие страницы с расчётами:

Замечания и предложения принимаются и приветствуются!

Рассматривая цепь переменного тока, содержащую только индуктивное сопротивление (смотрите статью «Катушка индуктивности в цепи переменного тока»), мы предполагали равным нулю активное сопротивление этой цепи.

Однако в действительности как провод самой катушки, так и соединительные провода обладают хотя и небольшим, но активным сопротивлением, поэтому цепь неизбежно потребляет энергию источника тока.

Поэтому при определении общего сопротивления внешней цепи нужно складывать ее реактивное и активное сопротивления. Но складывать эти два различных по своему характеру сопротивления нельзя.

В этом случае полное сопротивление цепи переменному току находят путем геометрического сложения.

Строят прямоугольный треугольник (см. рисунок 1) одной стороной которого служит величина индуктивного сопротивления, а другой – величина активного сопротивления. Искомое полное сопротивление цепи определится третьей стороной треугольника.

Рисунок 1. Определение полного сопротивления цепи, содержащей индуктивное и активное сопротивление

Полное сопротивление цепи обозначается латинской буквой Z и измеряется в омах. Из построения видно, что полное сопротивление всегда больше индуктивного и активного сопротивлений, отдельно взятых.

Алгебраическое выражение полного сопротивления цепи имеет вид:

где Z — общее сопротивление, R — активное сопротивление, X L — индуктивное сопротивление цепи.

Таким образом, полное сопротивление цепи переменному току, состоящей из активного и индуктивною сопротивлений, равно корню квадратному из суммы квадратов активного и индуктивного сопротивлений этой цепи.

Закон Ома для такой цепи выразится формулой I = U / Z , где Z — общее сопротивление цепи.

Разберем теперь, какое будет напряжение, если цепь, кроме и и сдвиг фаз между током и на индуктивности, обладает также сравнительно большим активным сопротивлением. На практике такой цепью может служить, например, цепь, содержащая катушку индуктивности без железного сердечника, намотанную из тонкой проволоки (дроссель высокой частоты).

В этом случае сдвиг фаз между током и напряжением составит уже не четверть периода (как это было в цепи только с индуктивным сопротивлением), а значительно меньше; причем чем больше будет активное сопротивление, тем меньший получится сдвиг фаз.

Рисунок 2. Ток и напряжение в цепи, содержащей R и L

Теперь и сама ЭДС самоиндукции не находится в противофазе с напряжением источника тока, так как сдвинута относительно напряжения уже не на половину периода, а меньше. Кроме того, напряжение, создаваемое источником тока на зажимах катушки, не равно ЭДС самоиндукции, а больше нее на величину падения напряжения в активном сопротивлении провода катушки. Иначе говоря, напряжение на катушке состоит как бы из двух слагающих:

uL— реактивной слагающей напряжения, уравновешивающей действие ЭДС самоиндукции,

uR — активной слагающей напряжения, идущей на преодоление активного сопротивления цепи.

Если бы мы включили в цепь последовательно с катушкой большое активное сопротивление, то сдвиг фаз настолько бы уменьшился, что синусоида тока почти догнала бы синусоиду напряжения и разность фаз между ними была бы едва заметна. В этом случае амплитуда слагающей и, была бы больше амплитуды слагающей.

Точно так же можно уменьшить сдвиг фаз и даже совсем свести его к нулю, если уменьшить каким-либо способом частоте генератора. Уменьшение частоты приведет к уменьшению ЭДС самоиндукции, а следовательно, и к уменьшению вызываемого ею сдвига фаз между током и напряжением в цепи.

Мощность цепи переменного тока, содержащей катушку индуктивности

Цепь переменного тока, содержащая катушку, не потребляет энергии источника тока и что в цепи происходит процесс обмена энергией между генератором и цепью.

Разберем теперь, как будет обстоять дело с мощностью, потребляемой такой цепью.

Мощность, потребляемая в цепи переменного тока, равна произведению тока на напряжение, но так как ток и напряжение есть переменные величины, то и мощность будет также переменной. При этом значение мощности для каждого момента времени мы сможем определить, если умножим величину тока на величину напряжения, соответствующую данному моменту времени.

Чтобы получить график мощности, мы должны перемножить величины отрезков прямых линий, определяющие ток и напряжение в различные моменты времени. Такое построение и приведено на рис. 3, а. Пунктирная волнообразная кривая р показывает нам, как изменяется мощность в цепи переменного тока, содержащей только индуктивное сопротивление.

При построении этой кривой использовалось следующее правило алгебраического умножения : при умножении положительной величины на отрицательную получается отрицательная величина, а при перемножении двух отрицательных или двух положительных — положительная величина.

Рисунок 3. Графики мощности: а – в цепи содержащей индуктивное сопротивление, б – тоже, активное сопротивление

Рисунок 4. График мощности для цепи, содержащей R и L

Кривая мощности в этом случае расположена выше оси времени. Это значит, что обмена энергией между генератором и цепью не происходит, а следовательно, мощность, отдаваемая генератором в цепь, полностью потребляется цепью.

На рис. 4 изображен график мощности для цепи, содержащей в себе одновременно индуктивное и активное сопротивления. В этом случае также происходит обратный переход энергии из цепи к источнику тока, однако в значительно меньшей степени, чем в цепи с одним индуктивным сопротивлением.

Рассмотрев приведенные выше графики мощности, мы приходим к выводу, что только сдвиг фаз между током и напряжением в цепи создает «отрицательную» мощность. При этом, чем больше будет сдвиг фаз между током и напряжением в цепи тем потребляемая цепью мощность будет меньше, и, наоборот, чем меньше сдвиг фаз, тем потребляемая цепью мощность будет больше.

Емкостное реактивное сопротивление — обзор

X

Символ реактивного сопротивления.

X C

Символ емкостного реактивного сопротивления.

X L

Символ индуктивного реактивного сопротивления.

Демодуляция X и Z

Система демодуляции цветного ТВ, в которой два повторно вставлены 3.Сигналы поднесущей 58 МГц различаются примерно на 60 °, а не на обычные 90 °. Напряжения R-Y, B-Y и G-Y выводятся из демодулированных сигналов, и эти напряжения управляют тремя пушками кинескопа. Важным преимуществом этой системы является то, что схема приемника проще, чем требуется при I- и Q-демодуляции.

Ось X

1. Референтная ось в кристалле кварца. 2. Горизонтальная ось в системе прямоугольных координат.3. Направление по горизонтали или слева направо в двумерной системе координат. X-X обозначает одно направление, которому следует следовать в методе пошагового повторения.

Диапазон X

Радиочастотный диапазон от 5200 до 11000 МГц с длинами волн от 5,77 до 2,75 см.

X-bar

Прямоугольный кристаллический стержень, обычно вырезанный из Z-образного сечения, вытянутый параллельно X и с краями, параллельными X, Y и Z.

X-конденсатор

Конденсатор для подавления радиопомех, предназначенный для применений, в которых отказ конденсатора не приведет к опасности поражения электрическим током.

Кристалл X-cut

Кристалл, вырезанный так, что его основные поверхности перпендикулярны электрической оси (X) исходного кристалла кварца.

ксенон

Инертный газ, используемый в некоторых тиратронах и других газовых трубках.

ксеноновая импульсная лампа

Источник некогерентного белого света высокой интенсивности; он работает, разряжая конденсатор через трубку с газом ксеноном. Такое устройство часто используется в качестве источника излучения накачки для различных лазеров с оптическим возбуждением.

ксерографический принтер

Устройство для печати оптического изображения на бумаге; светлые и темные области представлены электростатически заряженными и незаряженными участками на бумаге. Порошковые чернила, присыпанные пылью на бумаге, прилипают к заряженным участкам и впоследствии растворяются в бумаге под воздействием тепла.

ксерографическая запись

Ксерографическая запись.

ксерография

1.Эта ветвь электростатической электрофотографии, в которой изображения формируются на светопроводящей изолирующей среде с помощью инфракрасного, видимого или ультрафиолетового излучения. Затем среду присыпают порошком, который прилипает только к электростатически заряженному изображению. Затем применяется тепло, чтобы сплавить порошок в постоянное изображение. 2. Процесс печати электростатической электрофотографии, в котором используется фотопроводящая изолирующая среда в сочетании с инфракрасным, видимым или ультрафиолетовым излучением для создания структур скрытого электростатического заряда для достижения наблюдаемой записи.

xeroprinting

Та область электростатической электрофотографии, в которой узор из изоляционного материала на проводящей среде используется для формирования структур электростатического заряда для использования при копировании.

xeroradiography

Процесс печати электростатической электрофотографии, в котором используется светопроводящая изолирующая среда в сочетании с рентгеновскими или гамма-лучами для создания структур скрытого электростатического заряда для получения наблюдаемого рисунка.

ксерорадиографическое оборудование

Оборудование, использующее принципы электростатики и фотопроводимости для записи рентгеновских изображений на сенсибилизированную пластину через короткое время после экспонирования.

xfmr

Сокращенное обозначение трансформатора.

xistor

Сокращенное обозначение транзистора.

Разъем XLR

Экранированный трехжильный микрофонный штекер или розетка с фиксатором для отпирания пальцем для предотвращения случайного извлечения.Стандартный разъем для профессиональных пользователей микрофонов.

xmitter

Аббревиатура передатчика. Также сокращенно trans или xmtr.

xmsn

Сокращенное обозначение передачи.

xmtr

Аббревиатура передатчика. Также сокращенно транс или xmitter.

X-off

Передатчик выключен.

X-on

Передатчик включен.

X-частица

Частица, имеющая такой же отрицательный заряд, что и электрон, но масса между электроном и протоном. Он создается космическим излучением, падающим на молекулы газа или фактически составляющим часть космических лучей.

Рентгеновский аппарат

Рентгеновская трубка и принадлежности к ней, включая рентгеновский аппарат.

Рентгеновская кристаллография

1. Использование рентгеновских лучей для изучения расположения атомов в кристалле.2. Изучение структуры кристаллических материалов с использованием взаимодействия рентгеновских лучей и электронной плотности кристалла (дифракции).

Устройство для обнаружения рентгеновских лучей

Устройство, обнаруживающее неоднородности поверхности и объема твердых тел с помощью рентгеновских лучей.

Камера для дифракции рентгеновских лучей

Камера, которая направляет пучок рентгеновских лучей на образец неизвестного материала и позволяет полученным дифрагированным лучам воздействовать на полосу пленки.

Рентгенограмма

Картина, полученная на пленке, экспонированной с помощью рентгеновской дифракционной камеры. Он состоит из частей кругов с разным расстоянием между ними в зависимости от исследуемого материала.

Рентгеновский гониометр

Прибор, который определяет положение электрических осей кристалла кварца путем отражения рентгеновских лучей от атомных плоскостей кристалла.

Рентгеновские лучи

Также называются рентгеновскими лучами.Проникающее излучение похоже на свет, но имеет гораздо более короткие длины волн (от 10 –7 до 10 –10 см). Обычно они возникают при бомбардировке металлической мишени потоком высокоскоростных электронов.

Рентгеновский спектрограф

Прибор, который используется для построения диаграмм дифракции рентгеновских лучей, такой как рентгеновский спектрометр с фотографическими или другими регистрирующими устройствами.

Рентгеновский спектрометр

1.Прибор для получения рентгеновского спектра и измерения длин волн его компонентов. 2. Прибор, предназначенный для получения рентгеновского спектра материала в качестве помощи в его идентификации. Этот метод особенно полезен, когда материал не может быть физически разрушен.

Спектр рентгеновского излучения

Расположение пучка рентгеновских лучей в порядке длины волны.

Рентгеновский толщиномер

Бесконтактный толщиномер, используемый для измерения и индикации толщины движущегося холоднокатаного стального листа в процессе прокатки.Рентгеновский луч, направленный через лист, поглощается пропорционально толщине материала и его атомному номеру, и измерение количества поглощения дает непрерывное указание толщины листа.

Рентгеновская трубка

Вакуумная трубка, в которой рентгеновские лучи производятся путем бомбардировки мишени высокоскоростными электронами, ускоренными электростатическим полем.

Мишень для рентгеновской трубки

Также известен как антикатод.Электрод или электродная секция, на которую фокусируется электронный луч и излучает рентгеновские лучи.

xso

Аббревиатура для кварцевого стабилизатора.

xtal

Аббревиатура кристалла.

X-wave

Одна из двух составляющих, на которые магнитное поле Земли делит радиоволну в ионосфере. Другой компонент — обыкновенная, или О- волна.

Кристалл XY-огранки

Кристалл, ограненный таким образом, что его характеристики находятся между кристаллами X- и Y-огранки.

Плоттер XY

1. Устройство, используемое вместе с компьютером для нанесения координатных точек в виде графика. 2. Компьютерное устройство вывода, которое реагирует на цифровые сигналы предварительно записанных и / или обработанных данных путем распечатки линейных сегментов. Эти данные, которые могут включать буквенно-цифровые символы, диаграммы, таблицы или рисунки, загружаются из памяти компьютера со скоростью, достаточной для работы плоттера.XY-плоттер нельзя использовать для прямой записи аналоговых сигналов без дигитайзеров.

Регистратор XY

1. Регистратор, который отслеживает на диаграмме взаимосвязь между двумя переменными, ни одна из которых не является временем. Иногда диаграмма перемещается, и одна из переменных контролируется так, что взаимосвязь действительно увеличивается пропорционально времени. 2. Регистратор, в котором два сигнала одновременно записываются одним пером, которое приводится в движение в одном направлении (ось X) одним сигналом, а в другом направлении (ось Y) — вторым сигналом.3. Регистратор данных, который используется для записи изменения одного параметра по отношению к другому. Например, изменение давления с температурой. Для этих самописцев доступен широкий спектр преобразователей для преобразования физических параметров в электрические сигналы, которые можно использовать в самописце. Датчики давления, термопары, тензодатчики и акселерометры — вот несколько примеров. 4. Тип регистратора, который реагирует на поступающие аналоговые сигналы по мере их появления. Сигналы печатаются на графике заранее определенного размера, который может охватывать тестовые периоды от нескольких секунд до целого года.Регистратор XY записывает непрерывными линиями. Кроме того, скорость отклика прибора важна для точности записи.

Переключатель XY

Переключатель с дистанционным управлением, расположенный таким образом, что дворники перемещаются вперед и назад по горизонтали.

Калькулятор емкостного реактивного сопротивления

Это калькулятор емкостного реактивного сопротивления — отличный инструмент, который поможет вам оценить так называемое сопротивление конденсатора в электрической цепи.Вы можете найти формулу емкостного реактивного сопротивления в тексте ниже, и мы объясним, почему реактивное сопротивление возникает для переменного, а не постоянного тока. Если вы хотите узнать, как рассчитать емкостное реактивное сопротивление, вы попали в нужное место — поехали!

Что такое емкостное реактивное сопротивление?

Реактивное сопротивление — это свойство элемента электрической цепи противодействовать прохождению тока . Используя это определение, мы можем сказать, что емкостное сопротивление аналогично сопротивлению конденсатора .Даже единица реактивного сопротивления такая же, как и сопротивление — Ом ( Ом, ). Обычно мы обозначаем реактивное сопротивление как X .

Хотя и реактивное сопротивление ( X ), и сопротивление ( R ) имеют тенденцию быть одним и тем же в цепи, между ними существует определенное различие. Реактивное сопротивление влияет на переменный ток (AC), а сопротивление влияет на постоянный ток (DC) . Как правило, они являются составляющими импеданса Z , комплексной величины, определяющей полное противодействие цепи протеканию тока:

Z = R ± j * X ,

, где j = √-1 — мнимое число (квадратный корень из отрицательного числа).

Емкостное реактивное сопротивление — это свойство конденсатора . Точно так же индуктивное реактивное сопротивление — это свойство катушки индуктивности. Идеальный резистор имеет нулевое реактивное сопротивление, в то время как это чисто резистивный элемент. Напротив, идеальные конденсаторы и катушки индуктивности имеют нулевое сопротивление .

Итак, строго говоря, сопротивления конденсатора не существует. Обычно мы рассматриваем эту фразу как сокращение для определения емкостного реактивного сопротивления.

Как рассчитать емкостное реактивное сопротивление? Формула емкостного реактивного сопротивления

Как мы упоминали в предыдущем разделе, емкостное реактивное сопротивление — это свойство конденсатора, которое противодействует переменному току.То же самое верно для любого набора конденсаторов, который мы можем расположить последовательно или параллельно.

Одним из важнейших свойств переменного тока является его частота f . Мы можем рассчитать емкостное реактивное сопротивление X конденсатора C , используя следующее уравнение:

Х = 1 / (2 * π * f * C) .

В качестве альтернативы мы можем записать формулу емкостного реактивного сопротивления как:

Х = 1 / (ω * С) ,

, где ω = 2 * π * f — угловая частота тока.

Как видите, , чем выше частота , , емкость , , тем ниже реактивное сопротивление . Имеет ли это смысл?

Совершенно верно! Помните, что конденсатор накапливает электрическую энергию. Во время зарядки похоже, что конденсатор почти беспрепятственно пропускает ток. Чем больше он может поглотить (чем выше емкость), тем меньше он сопротивляется пропусканию тока. Кроме того, чем выше частота переменного тока, тем меньше времени остается для полной зарядки конденсатора.В случае постоянного тока ( f = 0 ) конденсатор сначала заряжается, но затем (в состоянии равновесия) он действует как разомкнутая цепь.

Как пользоваться калькулятором емкостного реактивного сопротивления?

Нет ничего сложного в оценке емкостного реактивного сопротивления любого конденсатора. Попрактикуемся в вычислениях на примере.

Допустим, у нас есть схема со сферическим конденсатором емкостью С = 30 нФ . Применяем источник напряжения, которое чередуется с частотой f = 60 Гц .Какое емкостное сопротивление в этой цепи?

  1. Перевести единицу емкости в Фарады. Мы можем использовать научную нотацию, чтобы записать значения компактно: C = 30 нФ = 3 · 10⁻⁸ F .

  2. Найдите произведение всех значений в знаменателе формулы емкостного реактивного сопротивления: 2 * π * f * C = 2 * π * 60 * 3 · 10⁻⁸ = 1,131 · 10⁻⁵ .

  3. Найдите его обратную мультипликативную величину, которая является отношением 1 и нашего произведения: 1/1.131 · 10⁻⁵ = 88 419,41 Ом . Не забывайте про единицу реактивного сопротивления!

  4. Запишите результат, используя соответствующий префикс: X = 88,41941 кОм .

  5. Округлим результат до четырех значащих цифр:

    X = 88,42 кОм .

  6. Проверьте результат с помощью нашего калькулятора емкостного реактивного сопротивления! Вау, относительно безболезненно, не так ли?

Цепи конденсаторов переменного тока | Реактивное сопротивление и импеданс — емкостный

Конденсаторы Vs.Резисторы

Конденсаторы не ведут себя так же, как резисторы. В то время как резисторы пропускают через себя поток электронов, прямо пропорциональный падению напряжения, конденсаторы противодействуют изменениям напряжения, потребляя или подавая ток во время зарядки или разрядки до нового уровня напряжения.

Поток электронов «через» конденсатор прямо пропорционален скорости изменения напряжения на конденсаторе. Это противодействие изменению напряжения представляет собой еще одну форму реактивного сопротивления , но оно прямо противоположно тому, которое демонстрируют индукторы.

Характеристики цепи конденсатора

Выражаясь математически, соотношение между током, протекающим через конденсатор, и скоростью изменения напряжения на конденсаторе выглядит следующим образом:

Выражение de / dt получено из расчетов, означающее скорость изменения мгновенного напряжения (e) во времени в вольтах в секунду. Емкость (C) выражается в фарадах, а мгновенный ток (i), конечно, выражается в амперах.

Иногда можно встретить скорость мгновенного изменения напряжения с течением времени, выраженную как dv / dt вместо de / dt: вместо напряжения используется строчная буква «v» или «e», но это означает то же самое. Чтобы показать, что происходит с переменным током, давайте проанализируем простую схему конденсатора:

Чистая емкостная цепь: напряжение конденсатора отстает от тока конденсатора на 90 °

Если бы мы изобразили ток и напряжение для этой очень простой схемы, это выглядело бы примерно так:

Формы сигналов чисто емкостной цепи.

Помните, что ток через конденсатор — это реакция на изменение напряжения на нем.

Следовательно, мгновенный ток равен нулю всякий раз, когда мгновенное напряжение находится на пике (нулевое изменение или наклон уровня синусоидальной волны напряжения), а мгновенный ток находится на пике везде, где мгновенное напряжение имеет максимальное изменение (точки крутизны на волне напряжения, где она пересекает нулевую линию).

Это приводит к появлению волны напряжения, сдвинутой по фазе на -90 ° по фазе с волной тока.Глядя на график, кажется, что волна тока имеет «фору» по сравнению с волной напряжения; ток «опережает» напряжение, а напряжение «отстает» от тока.

Напряжение отстает от тока на 90 ° в чисто емкостной цепи.

Как вы могли догадаться, та же необычная волна мощности, которую мы видели в простой цепи индуктивности, присутствует и в простой цепи конденсатора:

В чисто емкостной цепи мгновенная мощность может быть положительной или отрицательной.

Как и в случае с простой схемой индуктивности, сдвиг фазы на 90 градусов между напряжением и током приводит к появлению волны мощности, которая в равной степени чередуется между положительным и отрицательным. Это означает, что конденсатор не рассеивает мощность, поскольку он реагирует на изменения напряжения; он просто поочередно поглощает и высвобождает энергию.

Реактивное сопротивление конденсатора

Противодействие конденсатора изменению напряжения означает сопротивление переменному напряжению в целом, которое по определению всегда изменяется по мгновенной величине и направлению.

Для любой заданной величины переменного напряжения и заданной частоты конденсатор заданного размера будет «проводить» определенную величину переменного тока.

Точно так же, как ток через резистор является функцией напряжения на резисторе и сопротивления, предлагаемого резистором, переменный ток через конденсатор является функцией переменного напряжения на нем и реактивного сопротивления , обеспечиваемого конденсатором. .

Как и в случае катушек индуктивности, реактивное сопротивление конденсатора выражается в омах и обозначается буквой X (или, точнее, XC).

Поскольку конденсаторы «проводят» ток пропорционально скорости изменения напряжения, они будут пропускать больше тока при более быстром изменении напряжения (поскольку они заряжаются и разряжаются до тех же пиков напряжения за меньшее время) и меньший ток при более медленном изменении напряжения. .

Это означает, что реактивное сопротивление в Ом для любого конденсатора равно , обратно пропорционально частоте переменного тока.

Реактивное сопротивление конденсатора 100 мкФ:
Частота (Герцы) Реактивное сопротивление (Ом)
60 26.5258
120 13,2629
2500 0,6366

Обратите внимание, что отношение емкостного реактивного сопротивления к частоте прямо противоположно отношению индуктивного реактивного сопротивления.

Емкостное реактивное сопротивление (в омах) уменьшается с увеличением частоты переменного тока. И наоборот, индуктивное реактивное сопротивление (в омах) увеличивается с увеличением частоты переменного тока. Индукторы противодействуют более быстрому изменению токов, создавая большие падения напряжения; Конденсаторы противодействуют более быстрому изменению падений напряжения, допуская большие токи.

Как и в случае с индукторами, член 2πf в уравнении реактивного сопротивления может быть заменен строчной греческой буквой Омега (ω), которая обозначается как угловая скорость цепи переменного тока. Таким образом, уравнение XC = 1 / (2πfC) также может быть записано как XC = 1 / (ωC), с приведением ω в единицах радиан в секунду .

Переменный ток в простой емкостной цепи равен напряжению (в вольтах), деленному на емкостное реактивное сопротивление (в омах), точно так же, как переменный или постоянный ток в простой резистивной цепи равен напряжению (в вольтах), деленному на сопротивление (в Ом).Следующая схема иллюстрирует это математическое соотношение на примере:

Емкостное реактивное сопротивление.

Однако нужно иметь в виду, что здесь напряжение и ток не совпадают по фазе. Как было показано ранее, ток имеет фазовый сдвиг + 90 ° по отношению к напряжению. Если мы математически представим эти фазовые углы напряжения и тока, мы сможем вычислить фазовый угол реактивного сопротивления конденсатора току.

Напряжение в конденсаторе отстает от тока на 90 °.

Математически мы говорим, что фазовый угол сопротивления конденсатора току составляет -90 °, что означает, что сопротивление конденсатора току является отрицательной мнимой величиной. (См. Рисунок выше). Этот фазовый угол реактивного противодействия току становится критически важным при анализе цепей, особенно для сложных цепей переменного тока, где реактивное сопротивление и сопротивление взаимодействуют.

Будет полезно представить любое сопротивление компонента току в виде комплексных чисел, а не только скалярных величин сопротивления и реактивного сопротивления.

ОБЗОР:

  • Емкостное реактивное сопротивление — это противодействие, которое конденсатор предлагает переменному току из-за его сдвинутого по фазе накопления и выделения энергии в его электрическом поле. Реактивное сопротивление обозначается заглавной буквой «X» и измеряется в омах, как и сопротивление (R).
  • Емкостное реактивное сопротивление можно рассчитать по следующей формуле: XC = 1 / (2πfC)
  • Емкостное реактивное сопротивление уменьшается с увеличением частоты. Другими словами, чем выше частота, тем меньше он противодействует (тем больше «проводит») переменному току.

СВЯЗАННЫЕ РАБОЧИЕ ЛИСТЫ:

Реактивное сопротивление, индуктивное и емкостное | Физика II

Цели обучения

К концу этого раздела вы сможете:

  • Зависимость напряжения и тока от времени в простых индуктивных, емкостных и резистивных цепях.
  • Рассчитайте индуктивное и емкостное сопротивление.
  • Рассчитайте ток и / или напряжение в простых индуктивных, емкостных и резистивных цепях.

Многие цепи также содержат конденсаторы и катушки индуктивности в дополнение к резисторам и источнику переменного напряжения. Мы видели, как конденсаторы и катушки индуктивности реагируют на постоянное напряжение при его включении и выключении. Теперь мы исследуем, как катушки индуктивности и конденсаторы реагируют на синусоидальное переменное напряжение.

Катушки индуктивности и индуктивное сопротивление

Предположим, индуктор подключен непосредственно к источнику переменного напряжения, как показано на рисунке 1.Разумно предположить, что сопротивление пренебрежимо мало, поскольку на практике мы можем сделать сопротивление катушки индуктивности настолько малым, что оно окажет незначительное влияние на схему. Также показан график зависимости напряжения и тока от времени.

Рис. 1. (a) Источник переменного напряжения, включенный последовательно с катушкой индуктивности, имеющей незначительное сопротивление. (б) График зависимости тока и напряжения на катушке индуктивности от времени.

График на Рисунке 1 (b) начинается с максимального напряжения. Обратите внимание, что ток начинается с нуля и повышается до своего пика после напряжения, которое им управляет, как это было в случае, когда напряжение постоянного тока было включено в предыдущем разделе.Когда напряжение становится отрицательным в точке а, ток начинает уменьшаться; оно становится нулевым в точке b, где напряжение является самым отрицательным. Затем ток становится отрицательным, снова вслед за напряжением. Напряжение становится положительным в точке c и начинает делать ток менее отрицательным. В точке d ток проходит через ноль, когда напряжение достигает своего положительного пика, чтобы начать следующий цикл. Кратко это поведение можно описать следующим образом:

Напряжение переменного тока в индукторе

Когда на катушку индуктивности подается синусоидальное напряжение, оно опережает ток на одну четверть цикла или на фазовый угол 90º.

Ток отстает от напряжения, поскольку индукторы препятствуют изменению тока. При изменении тока возникает обратная ЭДС В = — L I / Δ t ). Это считается эффективным сопротивлением катушки индуктивности переменному току. Среднеквадратичный ток I через катушку индуктивности L определяется версией закона Ома:

[латекс] I = \ frac {V} {{X} _ {L}} \\ [/ latex],

, где В, — среднеквадратичное значение напряжения на катушке индуктивности, а X L определяется как

[латекс] {X} _ {L} = 2 \ pi {fL} \\ [/ латекс],

с f частота источника переменного напряжения в герцах (анализ схемы с использованием правила петли Кирхгофа и вычислений фактически дает это выражение). X L называется индуктивным реактивным сопротивлением , потому что катушка индуктивности препятствует прохождению тока. X L имеет единицы измерения Ом (1 Гн = 1 Ом с, так что частота, умноженная на индуктивность, имеет единицы (циклы / с) (Ом ⋅ с) = Ом)), что соответствует его роли в качестве эффективное сопротивление. Логично, что X L пропорционально L , поскольку чем больше индукция, тем больше сопротивление изменению.Также разумно, что X L пропорционально частоте f , поскольку большая частота означает большее изменение тока. То есть Δ I / Δ t является большим для больших частот (большие f , маленькие Δ t ). Чем больше изменение, тем больше сопротивление катушки индуктивности.

Пример 1. Расчет индуктивного сопротивления, а затем тока

(a) Вычислите индуктивное сопротивление 3.Индуктор 00 мГн при подаче переменного напряжения 60,0 Гц и 10,0 кГц. (b) Каков среднеквадратичный ток на каждой частоте, если приложенное действующее напряжение составляет 120 В?

Стратегия

Индуктивное реактивное сопротивление находится непосредственно из выражения X L = 2πf L . Как только X L было найдено на каждой частоте, закон Ома, как указано в уравнении I = V / X L , можно использовать для определения тока на каждой частоте.

Решение для (a)

Ввод частоты и индуктивности в уравнение X L = 2πf L дает

X L = 2πf L = 6,28 (60,0 / с) (3,00 мГн) = 1,13 Ом при 60 Гц.

Аналогично, на 10 кГц,

X L = 2πf L = 6,28 (1,00 × 10 4 / с) (3,00 мГн) = 188 Ом при 10 кГц.

Решение для (b)

Среднеквадратичное значение тока теперь определяется с использованием версии закона Ома в уравнении I = В / X L , при условии, что приложенное действующее напряжение составляет 120 В.Для первой частоты это дает

[латекс] I = \ frac {V} {{X} _ {L}} = \ frac {120 \ text {V}} {1.13 \ text {} \ Omega} = 106 \ text {A at} 60 \ текст {Hz} \\ [/ latex].

Аналогично, на 10 кГц,

[латекс] I = \ frac {V} {{X} _ {L}} = \ frac {120 \ text {V}} {188 \ text {} \ Omega} = 0,637 \ text {A at} 10 \ текст {кГц} \\ [/ latex].

Обсуждение

Катушка индуктивности по-разному реагирует на двух разных частотах. На более высокой частоте его реактивное сопротивление велико, а ток невелик, что соответствует тому, как катушка индуктивности препятствует быстрому изменению.Таким образом, наиболее затруднены высокие частоты. Индукторы могут использоваться для фильтрации высоких частот; например, большую катушку индуктивности можно включить последовательно с системой воспроизведения звука или последовательно с вашим домашним компьютером, чтобы уменьшить высокочастотный звук, выводимый из ваших динамиков или высокочастотные всплески мощности на ваш компьютер.

Обратите внимание, что, хотя сопротивлением в рассматриваемой цепи можно пренебречь, переменный ток не очень велик, поскольку индуктивное реактивное сопротивление препятствует его протеканию.С переменным током нет времени, чтобы ток стал слишком большим.

Конденсаторы и емкостное сопротивление

Рассмотрим конденсатор, подключенный непосредственно к источнику переменного напряжения, как показано на рисунке 2. Сопротивление такой цепи можно сделать настолько малым, что оно окажет незначительное влияние по сравнению с конденсатором, поэтому мы можем предположить, что сопротивление незначительно. Напряжение на конденсаторе и ток показаны на рисунке как функции времени.

Рисунок 2.(а) Источник переменного напряжения, включенный последовательно с конденсатором С, имеющим незначительное сопротивление. (б) График зависимости тока и напряжения на конденсаторе от времени.

График на рисунке 2 начинается с максимального напряжения на конденсаторе. В этот момент ток равен нулю, потому что конденсатор полностью заряжен и останавливает поток. Затем напряжение падает, а ток становится отрицательным по мере разряда конденсатора. В точке а конденсатор полностью разряжен (на нем Q = 0 ) и напряжение на нем равно нулю.Ток остается отрицательным между точками a и b, вызывая обратное напряжение на конденсаторе. Это завершается в точке b, где ток равен нулю, а напряжение имеет самое отрицательное значение. Ток становится положительным после точки b, нейтрализуя заряд конденсатора и доводя напряжение до нуля в точке c, что позволяет току достичь своего максимума. Между точками c и d ток падает до нуля, когда напряжение достигает своего пика, и процесс начинает повторяться. На протяжении всего цикла напряжение соответствует тому, что делает ток, на одну четверть цикла:

Напряжение переменного тока в конденсаторе

Когда на конденсатор подается синусоидальное напряжение, оно следует за током на одну четверть цикла или на фазовый угол 90º.

Конденсатор влияет на ток, имея возможность полностью его отключить, когда он полностью заряжен. Поскольку применяется переменное напряжение, возникает среднеквадратичный ток, но он ограничивается конденсатором. Это считается эффективным сопротивлением конденсатора переменному току, поэтому среднеквадратичный ток I в цепи, содержащей только конденсатор C , определяется другой версией закона Ома как

[латекс] I = \ frac {V} {{X} _ {C}} \\ [/ latex],

, где В, — среднеквадратичное напряжение, а X C определяется (Как и в случае с X L , это выражение для X C является результатом анализа цепи используя правила и исчисление Кирхгофа) равным

[латекс] {X} _ {C} = \ frac {1} {2 \ pi fC} \\ [/ latex],

, где X C называется емкостным реактивным сопротивлением , потому что конденсатор препятствует прохождению тока. X C имеет единицы измерения Ом (проверка оставлена ​​в качестве упражнения для читателя). X C обратно пропорционально емкости C ; Чем больше конденсатор, тем больший заряд он может накапливать и тем больше может протекать ток. Она также обратно пропорциональна частоте f ; чем выше частота, тем меньше времени остается для полной зарядки конденсатора, и поэтому он меньше препятствует току.

Пример 2. Расчет емкостного реактивного сопротивления, а затем тока

(a) Рассчитайте емкостное реактивное сопротивление конденсатора 5,00 мФ при приложении переменного напряжения 60,0 Гц и 10,0 кГц. (b) Каков среднеквадратичный ток, если приложенное действующее напряжение составляет 120 В?

Стратегия

Емкостное реактивное сопротивление находится непосредственно из выражения в [latex] {X} _ {C} = \ frac {1} {2 \ pi fC} \\ [/ latex]. После того, как X C было обнаружено на каждой частоте, закон Ома сформулирован как I = V / X C , чтобы найти ток на каждой частоте.

Решение для (a)

Ввод частоты и емкости в [латекс] {X} _ {C} = \ frac {1} {2 \ pi fC} \\ [/ latex] дает

[латекс] \ begin {array} {lll} {X} _ {C} & = & \ frac {1} {2 \ pi fC} \\ & = & \ frac {1} {6.28 \ left (60.0 / \ text {s} \ right) \ left (5.00 \ text {} \ mu \ text {F} \ right)} = 531 \ text {} \ Omega \ text {at} 60 \ text {Hz} \ end {массив }\\[/латекс].

Аналогично, на 10 кГц,

[латекс] \ begin {array} {lll} {X} _ {C} & = & \ frac {1} {2 \ pi fC} = \ frac {1} {6.{4} / \ text {s} \ right) \ left (5,00 \ mu \ text {F} \ right)} \\ & = & 3,18 \ text {} \ Omega \ text {at} 10 \ text {кГц} \ end {array} \\ [/ latex].

Решение для (b)

Действующее значение тока теперь определяется с использованием версии закона Ома в I = В / X C , учитывая приложенное действующее напряжение 120 В. Для первой частоты это дает

[латекс] I = \ frac {V} {{X} _ {C}} = \ frac {120 \ text {V}} {531 \ text {} \ Omega} = 0,226 \ text {A at} 60 \ текст {Hz} \\ [/ latex].

Аналогично, на 10 кГц,

[латекс] I = \ frac {V} {{X} _ {C}} = \ frac {120 \ text {V}} {3.18 \ text {} \ Omega} = 3.37 \ text {A at} 10 \ текст {Hz} \\ [/ latex].

Обсуждение

Конденсатор очень по-разному реагирует на двух разных частотах, а индуктор реагирует прямо противоположным образом. На более высокой частоте его реактивное сопротивление мало, а ток велик. Конденсаторы одобряют изменения, тогда как индукторы противодействуют изменениям. Конденсаторы больше всего препятствуют низким частотам, поскольку низкая частота позволяет им успеть зарядиться и остановить ток.Конденсаторы можно использовать для фильтрации низких частот. Например, конденсатор, включенный последовательно с системой воспроизведения звука, избавляет ее от гула 60 Гц.

Хотя конденсатор в основном представляет собой разомкнутую цепь, в цепи с напряжением переменного тока, приложенным к конденсатору, присутствует среднеквадратичный ток. Это связано с тем, что напряжение постоянно меняет направление, заряжая и разряжая конденсатор. Если частота стремится к нулю (DC), X C стремится к бесконечности, и ток равен нулю после зарядки конденсатора.На очень высоких частотах реактивное сопротивление конденсатора стремится к нулю — он имеет незначительное реактивное сопротивление и не препятствует току (действует как простой провод). Конденсаторы оказывают противоположное влияние на цепи переменного тока, чем индукторы .

Резисторы в цепи переменного тока

В качестве напоминания рассмотрим Рисунок 3, на котором показано напряжение переменного тока, приложенное к резистору, и график зависимости напряжения и тока от времени. Напряжение и ток равны в фазе в резисторе.Отсутствует частотная зависимость поведения простого сопротивления в цепи:

Рис. 3. (a) Источник переменного напряжения, включенный последовательно с резистором. (b) График зависимости тока и напряжения на резисторе от времени, показывающий, что они точно совпадают по фазе.

Напряжение переменного тока на резисторе

Когда на резистор подается синусоидальное напряжение, оно точно совпадает по фазе с током — они имеют фазовый угол 0 °.

Сводка раздела

  • Для катушек индуктивности в цепях переменного тока мы обнаруживаем, что когда на индуктор подается синусоидальное напряжение, напряжение опережает ток на одну четверть цикла или на фазовый угол 90 °.
  • Сопротивление катушки индуктивности изменению тока выражается как сопротивление переменному току.
  • Закон Ома для индуктора

    [латекс] I = \ frac {V} {{X} _ {L}} \\ [/ latex],

    , где В — действующее значение напряжения на катушке индуктивности.

  • X L определяется как индуктивное реактивное сопротивление, определяемое по формуле

    [латекс] {X} _ {L} = 2 \ pi fL \\ [/ латекс],

    с f частота источника переменного напряжения в герцах.

  • Индуктивное реактивное сопротивление X L выражается в единицах Ом и имеет наибольшее значение на высоких частотах.
  • Для конденсаторов мы обнаруживаем, что когда на конденсатор подается синусоидальное напряжение, напряжение следует за током на одну четверть цикла или на фазовый угол 90º.
  • Поскольку конденсатор может останавливать ток при полной зарядке, он ограничивает ток и предлагает другую форму сопротивления переменному току; Закон Ома для конденсатора

    [латекс] I = \ frac {V} {{X} _ {C}} \\ [/ latex],

    , где В — действующее значение напряжения на конденсаторе.

  • X C определяется как емкостное реактивное сопротивление, определяемое по формуле

    [латекс] {X} _ {C} = \ frac {1} {2 \ pi fC} \\ [/ latex].

  • X C имеет единицы измерения Ом и имеет наибольшее значение на низких частотах.

Концептуальные вопросы

1. Пресбиакузис — это возрастная потеря слуха, которая постепенно влияет на высокие частоты. Усилитель слухового аппарата предназначен для равномерного усиления всех частот. Чтобы отрегулировать его мощность на пресбиакузис, включите ли вы конденсатор последовательно или параллельно динамику слухового аппарата? Объяснять.

2. Будете ли вы использовать большую индуктивность или большую емкость последовательно с системой для фильтрации низких частот, таких как гул 100 Гц в звуковой системе? Объяснять.

3. Высокочастотный шум в сети переменного тока может повредить компьютеры. Использует ли съемный блок, предназначенный для предотвращения этого повреждения, большую индуктивность или большую емкость (последовательно с компьютером) для фильтрации таких высоких частот? Объяснять.

4. Зависит ли индуктивность от тока, частоты или и того, и другого? А как насчет индуктивного сопротивления?

5. Объясните, почему конденсатор на рисунке 4 (a) действует как фильтр низких частот между двумя цепями, тогда как конденсатор на рисунке 4 (b) действует как фильтр высоких частот.

Рисунок 4. Конденсаторы и катушки индуктивности. Конденсатор с высокой и низкой частотой.

6. Если конденсаторы на рис. 4 заменить катушками индуктивности, что будет действовать как фильтр низких частот, а какой — как фильтр высоких частот?

Задачи и упражнения

1. На какой частоте индуктор 30,0 мГн будет иметь реактивное сопротивление 100 Ом?

2. Какое значение индуктивности следует использовать, если требуется реактивное сопротивление 20,0 кОм при частоте 500 Гц?

3.Какую емкость следует использовать для получения реактивного сопротивления 2,00 МОм при 60,0 Гц?

4. На какой частоте конденсатор 80,0 мФ будет иметь реактивное сопротивление 0,250 Ом?

5. (a) Найдите ток через катушку индуктивности 0,500 H, подключенную к источнику переменного тока 60,0 Гц, 480 В. (б) Каким будет ток на частоте 100 кГц?

6. (a) Какой ток течет, когда источник переменного тока 60,0 Гц, 480 В подключен к конденсатору 0,250 мкФ? (b) Каким будет ток на частоте 25,0 кГц?

7. А 20.Источник 0 кГц, 16,0 В, подключенный к катушке индуктивности, вырабатывает ток 2,00 А. Что такое индуктивность?

8. Источник 20,0 Гц, 16,0 В вырабатывает ток 2,00 мА при подключении к конденсатору. Какая емкость?

9. (a) Катушка индуктивности, предназначенная для фильтрации высокочастотного шума от источника питания персонального компьютера, включается последовательно с компьютером. Какая минимальная индуктивность должна обеспечивать реактивное сопротивление 2,00 кОм для шума 15,0 кГц? (б) Каково его реактивное сопротивление при 60?0 Гц?

10. Конденсатор на рисунке 4 (а) предназначен для фильтрации низкочастотных сигналов, препятствуя их передаче между цепями. (а) Какая емкость необходима для создания реактивного сопротивления 100 кОм при частоте 120 Гц? (b) Каким будет его реактивное сопротивление на частоте 1,00 МГц? (c) Обсудите значение ваших ответов на (a) и (b).

11. Конденсатор на рисунке 4 (b) будет фильтровать высокочастотные сигналы, замыкая их на землю / землю. (a) Какая емкость необходима для получения реактивного сопротивления [латекса] \ text {10.0 м \ Omega} [/ latex] для сигнала 5,00 кГц? (б) Каким будет его реактивное сопротивление при 3,00 Гц? (c) Обсудите значение ваших ответов на (a) и (b).

12. Необоснованные результаты При регистрации напряжений, обусловленных мозговой активностью (ЭЭГ), на конденсатор подается сигнал 10,0 мВ с частотой 0,500 Гц, производящий ток 100 мА. Сопротивление незначительное. а) Какая емкость? б) Что неразумного в этом результате? (c) Какое предположение или предпосылка ответственны?

13. Создайте свою проблему Рассмотрите возможность использования индуктора последовательно с компьютером, работающим от электричества 60 Гц. Постройте задачу, в которой вы вычисляете относительное снижение напряжения входящего высокочастотного шума по сравнению с напряжением 60 Гц. Среди вещей, которые следует учитывать, — допустимое последовательное реактивное сопротивление катушки индуктивности для мощности 60 Гц и вероятные частоты шума, проходящего через линии электропередач.

Глоссарий

индуктивное сопротивление:
сопротивление катушки индуктивности изменению тока; вычисляется по X L = 2π fL
емкостное реактивное сопротивление:
сопротивление конденсатора изменению тока; рассчитывается по [latex] {X} _ {C} = \ frac {1} {2 \ pi fC} \\ [/ latex]

Избранные решения проблем и упражнения

1.531 Гц

3. 1,33 нФ

5. (а) 2,55 А (б) 1,53 мА

7. 63,7 мкГн

9. (а) 21,2 мГн (б) 8,00 Ом

Расчет емкостного реактивного сопротивления

  • Изучив этот раздел, вы сможете описать:
  • • Расчеты с учетом емкостного реактивного сопротивления.
  • • Многоступенчатые расчеты реактивного сопротивления и сопротивления.

Для расчетов, основанных на емкостном реактивном сопротивлении, вам сначала нужно подумать об информации на странице «Реактивное сопротивление и сопротивление» и оценить разницу между реактивным сопротивлением и сопротивлением и двумя типами реактивного сопротивления. Для расчетов вы можете выбрать, какую формулу использовать для емкостного реактивного сопротивления: 1 / 2πƒC или 1 / ωC, но чаще используется 1 / 2πƒC (одна из причин состоит в том, что в научных калькуляторах чаще всего есть клавиша π, но нет клавиши ω!) .

Помните, что некоторые проблемы, которые вам, возможно, придется решить, не обязательно имеют очевидное решение, например, простой расчет реактивного сопротивления компонента.Например, если вас просят рассчитать напряжение питания, необходимое для создания определенного тока, протекающего через компонент, или напряжение на компоненте, могут потребоваться два или более шагов, используя ответ из одного расчета, чтобы предоставить информацию для второго расчета до достижения окончательный ответ.

Прежде чем начать, подумайте о следующих советах; они облегчат задачу, если вы будете внимательно им следовать.

1. Разработайте ответы с помощью карандаша и бумаги; перерисуйте схему, над которой работаете.

2. Перечислите элементы информации, которую вам дают, и то, что вам нужно найти для своего ответа. Это поможет вам решить, можно ли найти ответ за один шаг или вам понадобится промежуточный ответ.

3. После того, как вы перечислили информацию на шаге 2, вам нужно будет решить, какую соответствующую формулу (или формулы) использовать. Запишите и это.

4. Конечно, ответ — это не просто число, если это определенное количество Ом (или любая другая единица измерения), не забудьте указать правильную единицу, кратную (например.грамм. Ω, KΩ или MΩ) в противном случае ваш ответ не имеет смысла.

5. Когда вы вводите значения в калькулятор, преобразуйте все большие или малые (мег, микро и т. Д.) Значения в их основные единицы (вольт-омы и т. Д.) С помощью клавиши EXP. Здесь легко ошибиться и получить действительно глупые ответы, в тысячи раз слишком большие или слишком маленькие.

Все эти шаги сначала могут показаться довольно утомительными, но войдите в привычку, и они упростят ваши вычисления, потому что вы будете следовать знакомому методу.Они также будут более надежными, потому что, когда вам нужно выполнить многоэтапные расчеты, вам нужно быть организованным. Так легко ошибиться на полпути в тренировке, потому что вы забыли, где именно вы находитесь в расчетах. Однако если вы выписали каждую проблему, это позволит вам вернуться назад и посмотреть, где вы ошиблись, чтобы не повторять одни и те же ошибки.

Зачем тратить время на все эти хлопоты, когда в Интернете есть множество калькуляторов, которые сделают вычисления за вас?

Многие электронные и сетевые калькуляторы превосходны, просто введите данные и нажмите, чтобы получить ответ.Но вам все равно нужно инстинктивно знать, какую формулу использовать, когда и почему. Чтобы быть достаточно знакомым, чтобы делать это хорошо, вам необходимо знать, как работают различные формулы. Лучший способ сделать это — начать с решения некоторых проблем вручную, тогда вы обнаружите, что многие из калькуляторов, предлагаемых на веб-сайтах, намного более полезны.

Чтобы помочь вам на правильном пути, почему бы не загрузить нашу брошюру «Подсказки по математике», в которой показано, как использовать калькулятор с экспонентами и инженерными обозначениями, чтобы работать с этими единицами и каждый раз получать правильный ответ.

Нет научного калькулятора? Буклет «Подсказки по математике» объясняет, что вам нужно (и что вам не нужно, чтобы не тратить деньги без надобности). Если вы не хотите покупать научный калькулятор, вы всегда можете получить его бесплатно на сайте www.calculator.org/download.html. Пользователи ПК могут попробовать Calc98.

Какой бы калькулятор вы ни выбрали, помните, что вам следует прочитать инструкции, чтобы ознакомиться с методами работы, которые вы должны использовать, поскольку они варьируются от калькулятора к калькулятору.

Хорошо, теперь вы прочитали эти инструкции, и вы готовы к работе. Вот способ решить типичную проблему на бумаге, чтобы (со временем) вы не запутались.

Рис. 6.4.1 Пример емкостного реактивного сопротивления

Примеры реактивного сопротивления.

Проблема, проиллюстрированная на рис. 6.4.1, является типичным примером, когда необходимо найти ряд связанных значений, включая реактивное сопротивление конденсатора. Другие значения, такие как действующее значение напряжения (V RMS ) и среднеквадратичное значение тока (I RMS ), описаны в Модуле 1.2

Примечание. Если вы используете Calc98 для своих расчетов, вам необходимо установить в меню View> Option> Display значение Engineering (в разделе «Decimal»), и было бы неплохо, пока вы находитесь в этом меню, выбрать 2 из В раскрывающемся списке Десятичные дроби можно указать количество цифр после десятичного разряда. Это округлит ваш ответ до двух десятичных знаков, что является достаточно точным для большинства применений и не даст вам получить глупые ответы, такие как 75.666666666667Ω, что было бы слишком точным для большинства целей.

онлайн-курсов PDH. PDH для профессиональных инженеров. ПДХ Инжиниринг.

«Мне нравится широта ваших курсов по HVAC; не только экологичность или экономия энергии

курс.

Russell Bailey, P.E.

Нью-Йорк

«Он укрепил мои текущие знания и научил меня еще нескольким новым вещам.

, чтобы познакомить меня с новыми источниками

информации.»

Стивен Дедак, П.Е.

Нью-Джерси

«Материал был очень информативным и организованным. Я многому научился, и они были

.

очень быстро отвечает на вопросы.

Это было на высшем уровне. Будет использовать

снова . Спасибо. «

Blair Hayward, P.E.

Альберта, Канада

«Простой в использовании веб-сайт.Хорошо организовано. Я действительно буду снова пользоваться вашими услугами.

проеду по твоей компании

имя другим на работе. «

Roy Pfleiderer, P.E.

Нью-Йорк

«Справочные материалы были превосходными, и курс был очень информативным, особенно потому, что я думал, что я уже знаком

с деталями Канзас

Городская авария Хаятт.»

Майкл Морган, P.E.

Техас

«Мне очень нравится ваша бизнес-модель. Мне нравится просматривать текст перед покупкой. Я нашел класс

.

информативно и полезно

на моей работе »

Вильям Сенкевич, П.Е.

Флорида

«У вас большой выбор курсов, а статьи очень информативны.Вы

— лучшее, что я нашел ».

Russell Smith, P.E.

Пенсильвания

«Я считаю, что такой подход позволяет работающему инженеру легко зарабатывать PDH, давая время на просмотр

материал «.

Jesus Sierra, P.E.

Калифорния

«Спасибо, что разрешили мне просмотреть неправильные ответы.На самом деле

человек узнает больше

от отказов »

John Scondras, P.E.

Пенсильвания

«Курс составлен хорошо, и использование тематических исследований является эффективным.

способ обучения »

Джек Лундберг, P.E.

Висконсин

«Я очень впечатлен тем, как вы представляете курсы; i.е., позволяя

студента для ознакомления с курсом

материала до оплаты и

получает викторину «

Арвин Свангер, П.Е.

Вирджиния

«Спасибо за то, что вы предложили все эти замечательные курсы. Я определенно выучил и

получил много удовольствия «.

Мехди Рахими, П.Е.

Нью-Йорк

«Я очень доволен предлагаемыми курсами, качеством материалов и простотой поиска.

в режиме онлайн

курса.»

Уильям Валериоти, P.E.

Техас

«Этот материал в значительной степени оправдал мои ожидания. По курсу было легко следовать. Фотографии в основном обеспечивали хорошее наглядное представление о

обсуждаемых тем ».

Майкл Райан, P.E.

Пенсильвания

«Именно то, что я искал. Потребовался 1 балл по этике, и я нашел его здесь.»

Джеральд Нотт, П.Е.

Нью-Джерси

«Это был мой первый онлайн-опыт получения необходимых мне кредитов PDH. Это было

информативно, выгодно и экономично.

Я очень рекомендую

всем инженерам.

Джеймс Шурелл, П.Е.

Огайо

«Я понимаю, что вопросы относятся к« реальному миру »и имеют отношение к моей практике, и

не на основании каких-то неясных раздел

законов, которые не применяются

«нормальная» практика.»

Марк Каноник, П.Е.

Нью-Йорк

«Отличный опыт! Я многому научился, чтобы перенести его на свой медицинский прибор.

организация.

Иван Харлан, П.Е.

Теннесси

«Материалы курса имели хорошее содержание, не слишком математическое, с хорошим акцентом на практическое применение технологий».

Юджин Бойл, П.E.

Калифорния

«Это был очень приятный опыт. Тема была интересной и хорошо изложенной,

а онлайн-формат был очень

доступный и простой

использовать. Большое спасибо. «

Патрисия Адамс, P.E.

Канзас

«Отличный способ добиться соответствия требованиям PE Continuing Education в рамках ограничений по времени лицензиата.»

Joseph Frissora, P.E.

Нью-Джерси

«Должен признать, я действительно многому научился. Помогает иметь распечатанный тест во время

обзор текстового материала. Я

также оценил просмотр

фактических случаев предоставлено.

Жаклин Брукс, П.Е.

Флорида

«Документ» Общие ошибки ADA при проектировании объектов «очень полезен.

испытание потребовало исследования в

документ но ответы были

в наличии. «

Гарольд Катлер, П.Е.

Массачусетс

«Я эффективно использовал свое время. Спасибо за широкий выбор вариантов.

в транспортной инженерии, что мне нужно

для выполнения требований

Сертификат ВОМ.»

Джозеф Гилрой, П.Е.

Иллинойс

«Очень удобный и доступный способ заработать CEU для моих требований PG в Делавэре».

Ричард Роудс, P.E.

Мэриленд

«Я многому научился с защитным заземлением. Пока все курсы, которые я прошел, были отличными.

Надеюсь увидеть больше 40%

курса со скидкой.»

Кристина Николас, П.Е.

Нью-Йорк

«Только что сдал экзамен по радиологическим стандартам и с нетерпением жду возможности сдать еще

курса. Процесс прост, и

намного эффективнее, чем

приходится путешествовать «.

Деннис Мейер, P.E.

Айдахо

«Услуги, предоставляемые CEDengineering, очень полезны для профессионалов

Инженеры получат блоки PDH

в любое время.Очень удобно ».

Пол Абелла, P.E.

Аризона

«Пока все отлично! Поскольку я постоянно работаю матерью двоих детей, у меня мало

время исследовать где к

получить мои кредиты от.

Кристен Фаррелл, П.Е.

Висконсин

«Это было очень познавательно и познавательно.Легко для понимания с иллюстрациями

и графики; определенно делает это

проще поглотить все

теории.

Виктор Окампо, P.Eng.

Альберта, Канада

«Хороший обзор принципов работы с полупроводниками. Мне понравилось пройти курс по

.

мой собственный темп во время моего утро

метро

на работу.»

Клиффорд Гринблатт, П.Е.

Мэриленд

«Просто найти интересные курсы, скачать документы и сдать

викторина. Я бы очень рекомендовал

вам на любой PE, требующий

CE единиц. «

Марк Хардкасл, П.Е.

Миссури

«Очень хороший выбор тем из многих областей техники.»

Randall Dreiling, P.E.

Миссури

«Я заново узнал то, что забыл. Я также рад оказать финансовую помощь

по ваш промо-адрес электронной почты который

пониженная цена

на 40%.

Конрадо Казем, П.E.

Теннесси

«Отличный курс по разумной цене. Воспользуюсь вашими услугами в будущем».

Charles Fleischer, P.E.

Нью-Йорк

«Это был хороший тест и фактически подтвердил, что я прочитал профессиональную этику

кодов и Нью-Мексико

правила. «

Брун Гильберт, П.E.

Калифорния

«Мне очень понравились занятия. Они стоили потраченного времени и усилий».

Дэвид Рейнольдс, P.E.

Канзас

«Очень доволен качеством тестовых документов. Буду использовать CEDengineerng

при необходимости дополнительных

аттестация. «

Томас Каппеллин, П.E.

Иллинойс

«У меня истек срок действия курса, но вы все же выполнили свое обязательство и дали

мне то, за что я заплатил — много

оценено! «

Джефф Ханслик, P.E.

Оклахома

«CEDengineering предлагает удобные, экономичные и актуальные курсы.

для инженера »

Майк Зайдл, П.E.

Небраска

«Курс был по разумной цене, а материалы были краткими, а

хорошо организовано.

Glen Schwartz, P.E.

Нью-Джерси

«Вопросы подходили для уроков, а материал урока —

.

хороший справочный материал

для деревянного дизайна.

Брайан Адамс, П.E.

Миннесота

«Отлично, я смог получить полезные рекомендации по простому телефонному звонку».

Роберт Велнер, P.E.

Нью-Йорк

«У меня был большой опыт работы в прибрежном строительстве — проектирование

Building курс и

очень рекомендую

Денис Солано, P.E.

Флорида

«Очень понятный, хорошо организованный веб-сайт. Материалы курса этики Нью-Джерси были очень хорошими

хорошо подготовлен. «

Юджин Брэкбилл, P.E.

Коннектикут

«Очень хороший опыт. Мне нравится возможность загружать учебные материалы на

.

обзор везде и

всякий раз, когда.»

Тим Чиддикс, P.E.

Колорадо

«Отлично! Сохраняю широкий выбор тем на выбор».

Уильям Бараттино, P.E.

Вирджиния

«Процесс прямой, без всякой ерунды. Хороший опыт».

Тайрон Бааш, П.E.

Иллинойс

«Вопросы на экзамене были зондирующими и демонстрировали понимание

материала. Полная

и комплексное.

Майкл Тобин, P.E.

Аризона

«Это мой второй курс, и мне понравилось то, что мне предложили курс

поможет по моей линии

работ.»

Рики Хефлин, P.E.

Оклахома

«Очень быстро и легко ориентироваться. Я обязательно воспользуюсь этим сайтом снова».

Анджела Уотсон, P.E.

Монтана

«Легко выполнить. Нет путаницы при подходе к сдаче теста или записи сертификата».

Кеннет Пейдж, П.E.

Мэриленд

«Это был отличный источник информации о солнечном нагреве воды. Информативный

и отличное освежение ».

Luan Mane, P.E.

Conneticut

«Мне нравится подход к регистрации и возможность читать материалы в автономном режиме, а затем

вернуться, чтобы пройти викторину «

Алекс Млсна, П.E.

Индиана

«Я оценил объем информации, предоставленной для класса. Я знаю

это вся информация, которую я могу

использовать в реальных жизненных ситуациях »

Натали Дерингер, P.E.

Южная Дакота

«Обзорные материалы и образец теста были достаточно подробными, чтобы я мог сделать

успешно завершено

курс.»

Ира Бродский, П.Е.

Нью-Джерси

«Веб-сайт прост в использовании, вы можете скачать материалы для изучения, а затем вернуться

и пройдите викторину. Очень

удобно а на моем

собственный график «

Майкл Глэдд, P.E.

Грузия

«Спасибо за хорошие курсы на протяжении многих лет.»

Деннис Фундзак, П.Е.

Огайо

«Очень легко зарегистрироваться, получить доступ к курсу, пройти тест и распечатать PDH

свидетельство. Спасибо за создание

процесс простой ».

Фред Шейбе, P.E.

Висконсин

«Опыт положительный.Быстро нашел курс, который соответствовал моим потребностям, и закончил

один час PDH в

один час. «

Стив Торкильдсон, P.E.

Южная Каролина

«Мне понравилось загружать документы для проверки содержания

и пригодность, до

имея платить за

материал

Ричард Вимеленберг, P.E.

Мэриленд

«Это хорошее напоминание об ЭЭ для инженеров, не занимающихся электричеством».

Дуглас Стаффорд, П.Е.

Техас

«Всегда есть возможности для улучшения, но я ничего не могу придумать в вашем

.

процесс, которому требуется

улучшение.»

Thomas Stalcup, P.E.

Арканзас

«Мне очень нравится удобство участия в викторине онлайн и получение сразу

сертификат. «

Марлен Делани, П.Е.

Иллинойс

«Учебные модули CEDengineering — очень удобный способ доступа к информации по

много разные технические зоны за пределами

по своей специализации без

приходится путешествовать.»

Гектор Герреро, П.Е.

Грузия

ac — Как получить формулу емкостного и индуктивного реактивного сопротивления

Уравнение, на котором я вырос, это:

Xc = 1 / (2 pi f C) выражено в омах

Если это уравнение, которое вы пытаетесь вывести, забудьте о нем … это уравнение не является (в математических терминах) «строго правильным», на самом деле это грубое «практическое правило», в котором есть некоторые встроенные предположения, что НИКОГДА не идентифицируются и нигде.Вы были на правильном пути, но не зашли достаточно далеко … Термин SIN wt / COS wt можно переписать как TAN wt, который оценивается как величина, которая изменяется от + бесконечности до -infinity, дважды в течение каждого цикла .

Уравнение, которое вы создали, фактически выражает МГНОВЕННОЕ СОПРОТИВЛЕНИЕ конденсатора, возбуждаемого синусоидой. (= мгновенное напряжение на конденсаторе, деленное на мгновенный ток, протекающий через конденсатор) Тот факт, что это значение (я назову его Rc) изменяется от + бесконечности до-бесконечности… дважды в течение каждого цикла … действительно на 100% правильно.

Мгновенное сопротивление оценивается как положительное значение Rc в течение 1-го и 3-го интервалов «1/4 цикла» управляющего сигнала синусоидальной волны, и оно оценивается как отрицательное значение Rc во 2-м и 4-м интервалах «1/4 цикла» та же форма управляющего сигнала.

Это просто означает, что энергия течет ВНУТРИ конденсатора в течение 1-го и 3-го (1/4 цикла) интервалов (т. Е. Цепь «заряжает» конденсатор = + Rc), а энергия течет ИЗ конденсатора во время 2-го и 4-й (1/4 цикла) интервалы.(т.е. колпачок «разряжает» энергию обратно в цепь = -Rc)

(повторяется, все это предполагает, что форма управляющего сигнала является синусоидальной … и ТОЛЬКО синусоидальной волной)

Тот факт, что значение Rc является полностью «диким» и изменяется от + бесконечности до-бесконечности … дважды … в течение каждого цикла … означает, что «сопротивление» конденсатора, приводимого в действие синусоидальной волной, НЕ ИМЕЕТ КОНКРЕТНОЕ ЗНАЧЕНИЕ, КОТОРОЕ МОЖНО ОПРЕДЕЛЯТЬ ИЛИ ИСПОЛЬЗОВАТЬ В РАСЧЕТАХ ЛЮБЫХ ЦЕПЕЙ. Другими словами, концепция СОПРОТИВЛЕНИЯ как выражения отношения напряжение / ток в конденсаторе (управляемом синусоидальной волной) БЕСПОЛЕЗНА.

Вот почему соотношение напряжение / ток конденсатора НИКОГДА не идентифицируется словом СОПРОТИВЛЕНИЕ … вместо этого «изобретается» НОВАЯ величина, которая похожа и гораздо более полезна … называется РЕАКТИВНОСТЬ, которая также выражается в Ом.

Реактивное сопротивление определяется как ОТНОШЕНИЕ МАКСИМАЛЬНОГО НАПРЯЖЕНИЯ к МАКСИМАЛЬНОМУ ТОКУ в каждом (приложенном) синусоидальном цикле … Для конденсатора максимальное НАПРЯЖЕНИЕ возникает при w = +1/4 цикла, когда SIN (w) = +1, а максимальный ток возникает при w = +0/4 цикла, когда COS (w) = +1.Подставив эти константы обратно в ваше уравнение, вы получите хорошо известное (основной алгебры) уравнение для емкостного реактивного сопротивления …

Xc = 1 / (2Pi f C)

Итак … это уравнение НЕ ВЕРНО в каждый момент времени … оно выражает отношение МАКСИМАЛЬНОГО напряжения к МАКСИМАЛЬНОМУ току, но игнорирует тот факт, что эти два максимума НЕ возникают одновременно …. и нет ничего в уравнении даже «намекнуть» на то, что делается этот (неэтичный) «трюк» … это ОЧЕНЬ объясняет …

Вот почему суммирование (простая алгебра) значений R и X должно выполняться с помощью сложения векторов, а не алгебраического сложения … векторы принимают во внимание «временные различия», когда суммируется … алгебраические суммы могут ‘ не делай этого.

Вы никогда, никогда не увидите подобное объяснение нигде в учебнике, потому что никто не хочет тратить время на объяснение всего этого … потому что это, по сути, большой беспорядок, который каждый хочет просто уйти … и оно исчезнет, ​​если вы воспользуетесь высшей математикой…. но «простые смертные» должны пробираться через жизнь с простой старой алгеброй, и в данном случае она просто не справится с этой задачей.

.

Добавить комментарий

Ваш адрес email не будет опубликован. Обязательные поля помечены *